Download A 34-year-old woman comes to the clinic because of left lower

Document related concepts

Postpartum infections wikipedia , lookup

Anovulation wikipedia , lookup

Cell-free fetal DNA wikipedia , lookup

Prenatal testing wikipedia , lookup

Transcript
A 34-year-old woman comes to the clinic because of left
lower quadrant pain for the last 4 months. The pain is
intermittent and seems to worsen during her periods. She
has no significant past medical history and does not take
any medications. She is sexually active with 1 partner and
they use condoms for birth control. She has never been
pregnant and has regular menstrual periods. Her last
menstrual period was 17 days ago. Physical examination and
vital signs are normal. Pelvic examination reveals mild
tenderness in the mid and left side of the pelvis. There is
no vaginal discharge or bleeding and the cervical os is
closed. The most appropriate next step is to
A. order a CT scan of the pelvis
B. order ultrasonography of the pelvis
C. prescribe nonsteroidal antiinflammatory
medications for menstrual cramps
D. prescribe oral contraceptives
E. schedule a diagnostic laparoscopy
Explanation:
The correct answer is B. The most likely cause of this
patient's left sided pelvic pain is ovarian cysts. A
follicular cyst occurs when the normal follicle does not
shrink after the release of the egg. Follicular cysts are
soft, have thin membrane walls, and contain clear fluid.
They can rupture, causing sudden, severe pain that
gradually goes away over several days. Usually, however,
they spontaneously disappear over the course of 1 or more
menstrual cycles. They can cause changes in periods and
pelvic pain. Depending on the size of the cyst, it can
just be observed for spontaneous resolution or treated
with oral contraceptive pills. The best way to confirm the
diagnosis of ovarian cysts is by ultrasonography of the
pelvis.
A CT of the pelvis (choice A) is not the best way to image
the ovaries or to confirm the diagnosis of ovarian cysts.
A CT does not provide the resolution that ultrasonography
does to evaluate the cyst in fine detail in order to
ensure that it is a simple cyst as opposed to a more
worrisome complex cyst.
Prescribing nonsteroidal antiinflammatory medications for
menstrual cramps (choice C) is not appropriate in the
management of this patient. She has had symptoms of
intermittent pelvic pain for the last 4 months which makes
menstrual cramps as the etiology of pain, less likely. It
is important to further evaluate the pelvis for a cause of
her symptoms and confirm the diagnosis of ovarian cysts.
It is also important to rule out neoplastic causes of pain
such as a cystadenoma or cystadenocarcinoma.
Prescribing oral contraceptive pills (choice D) is a
reasonable option once the diagnosis of an ovarian cyst is
1
made. However, it is important to make the diagnosis first
with ultrasonography of the pelvis. If the cyst appears
simple on ultrasonography and is 4 cm or less in size, it
can be followed with serial ultrasonography for
resolution. For simple uncomplicated ovarian cysts in the
premenopausal period, treatment with the administration of
high-dose monophasic oral contraceptive pills (>35 mcg
estradiol) is recommended after consultation with a
gynecologist to normalize cycles and to facilitate
reduction in cyst size.
Diagnostic laparoscopy (choice E) allows direct
visualization of the intraperitoneal contents including
the ovaries which can add diagnostic yield. Surgical
intervention and/or excision of relevant pathology may be
accomplished by this method. This procedure is not the
next appropriate step in the management of this patient.
Ultrasonography of the pelvis usually provides sufficient
information to make the diagnosis of simple uncomplicated
ovarian cysts, without the need for direct visualization
of the pelvic organs.
A 26-year-old woman, gravida 1, with insulin-dependent
diabetes mellitus comes to the office at 33-weeks
gestation, reporting decreased fetal movement for 1 day.
She is found to have poor glucose control, with serial
blood sugar levels greater than 150 mg/dL. The fetal heart
rate on the nipple stimulation test is non-reactive. The
baseline rate is 140/min, and late decelerations are
observed in the first 30 minutes of the test with each
contraction. The next step in management is
A.
B.
C.
D.
E.
a biophysical profile (BPP)
delivery
a nonstress test (NST)
a contraction stress test (CST)
a repeat nipple stimulation in 24 hours
Explanation:
The correct answer is A. Because there is a high false
positive rate with CSTs (up to 50% falsely abnormal), the
next step in the algorithm of fetal distress is to perform
a BPP. This test consists of an evaluation of the fetal
heart rate without any stressors (i.e., without an
oxytocic agent) along with four other parameters derived
from an obstetrical ultrasound within a given period of
time. The four parameters are gross body movement, fetal
tone, fetal breathing movement, and amniotic fluid index.
Each parameter is given 2 points for a maximum of 10
points. A score of 6 or below, necessitates immediate
delivery.
2
Delivery (choice B) in not indicated at this time, because
a positive CST has a high false positive rate. Therefore,
it would be premature to deliver an infant that is doing
well at such an early gestational age.
NST (choice C) is incorrect because an NST gives you even
less information about the status of the fetus than a CST.
The NST is just an observation of the fetal heart rate as
a function of time given without any stressors.
Contraction stress test (CST) (choice D) is incorrect. A
CST can be in response to endogenous oxytocic agent
(nipple stimulation test) or to an exogenous source such
at oxytocin (Pitocin). The intention of CST is to test
fetoplacental reserve by causing contractions of the
uterus and thus decreasing blood and oxygen supply to the
fetus. A negative CST is defined as no periodic
decelerations in the fetal heart rate with satisfactory
recordings and at least three adequate contractions. A
positive CST is defined as persistent and consistent late
decelerations of fetal heart rate noted with more than
half of adequate contractions. A negative CST is
reassuring and predicts fetal survival for up to 5-7 days
after the test. A positive CST, however, can be falsely
positive up to 50% of time.
Repeating the nipple stimulation in 24 hrs (choice E) is
incorrect. Waiting a day, may be too late and repeating
the nipple stimulation does not give you any extra
information about the fetal well-being.
You are the doctor on call in the well baby nursery at the
community hospital. One of the nurses calls you to ask
about one of your patients. The baby is now 30 hours old
and was born full term via vaginal delivery to a healthy
28-year-old mother. There were no complications at the
delivery and the baby has been feeding well. The nurse is
concerned that the baby looks "yellow". You ask her to send
for a bilirubin level. A few hours later she calls to tell
you that the total bilirubin level has come back at 18
mg/dL with a direct bilirubin level of 0.6 mg/dL. The
parents are now concerned about the baby's discoloration.
The most appropriate next step is to
A. reassure the mother that this is completely normal
and no additional studies or treatment are indicated
B. repeat the bilirubin level immediately as the
result must be a lab error
C. start phototherapy and repeat the bilirubin level
in 6 hours
D. transfer to the nearest neonatal intensive care
unit for an exchange transfusion
E. wait 6 hours and repeat the bilirubin level
3
Explanation:
The correct answer is C. Jaundice is a common occurrence
in the first week of life. "Physiologic jaundice" is
usually transient and is due to an increased bilirubin
load from increased red blood cell volume, decreased
survival time of the red blood cell, and an increased
enterohepatic circulation. Other factors include defective
hepatic uptake of bilirubin and inadequate conjugation of
bilirubin to bilirubin glucuronide for excretion. A normal
cord blood bilirubin level is about 1.5 mg/dl, and a
neonate's serum bilirubin normally increases by a maximum
of 5mg/dl in 24 hours to a maximum of about 15mg/dl at 3
days of life. A level of 18mg/dl at 30 hours of life is
not normal and requires phototherapy to bring the level
down. Phototherapy should bring the level down by 1-2mg/dl
within 4-6 hours. The level should be checked at that time
to ensure that the level has not continued to rise.
Besides phototherapy, an investigation into the cause for
the rapid rise of bilirubin should be undertaken. The most
important consideration is the blood type of the mother
and the blood type of the infant. A direct Coombs test
should also be performed. Other things to keep in mind
include a red cell defect such as an hemoglobinopathy,
polycythemia, extravascular blood loss, bacterial sepsis,
increased enterohepatic circulation (from an intestinal
obstruction), disorder of bilirubin metabolism, or an
endocrine disorder such as hypothyroidism.
Reassurance that this is completely normal (choice A) is
incorrect. This baby needs to be treated for
hyperbilirubinemia and more blood will need to be drawn to
follow the levels. The mother should be reassured that
this is a common occurrence, that is usually transient,
and has no future implications on the baby's development,
if treated appropriately.
Repeating the labs (choice B) is not correct because
although lab error is something to keep in mind when
interpreting all test results, this baby is jaundiced and
therefore it is unlikely to have a normal bilirubin level.
Jaundice usually becomes apparent at levels of 5mg/dl in
the neonate. As levels get higher, the jaundice usually
progresses from the face to the trunk and then down the
body.
An exchange transfusion (choice D) is not yet warranted in
this baby. If the level were greater than 25mg/dl or
started at greater than 20 mg/dl and did not improve with
phototherapy, this baby would require an exchange
transfusion. During this procedure, blood is removed from
the baby (usually from an umbilical artery catheter), and
the baby is transfused with cross matched blood through a
venous catheter. The exchange is done in 15ml increments.
There are multiple risks to the procedure including
thromboemboli, dysrhythmias, hyperkalemia, hypernatremia,
DIC, and transfusion reaction. And thus answer D is not
4
correct.
Wait 6 hours and repeat the level (choice E) is incorrect
because the bilirubin level is too high to allow you to
wait and watch. If the bilirubin level is allowed to rise
unchecked the baby is put at risk for kernicterus. When
the level of unconjugated bilirubin reaches higher than
20mg/dl in a full-term neonate, it can become neurotoxic
by accumulating in the basal ganglia, pons, or cerebellum
causing kernicterus. Clinically, it manifests as a variety
of neurologic symptoms ranging from lethargy and hypotonia
to severe encephalopathy and death. At the present rate of
rise, the bilirubin is likely to be higher than 20 mg/dlin
6 hours.
A 31-year-old woman comes to the emergency department
because of heavy vaginal bleeding. She is 10-weeks
pregnant, by her last menstrual period and has been getting
routine prenatal care. She has been in good health and her
pregnancy has been uneventful, until the heavy bleeding
started the night before. She reports having to change a
pad every hour and thinks she may even have passed blood
clots. She has no significant past medical history and her
only medication is a daily prenatal vitamin. She denies
alcohol, cigarette, or drug use. Vitals are normal. Pelvic
examination reveals an open cervical os and blood in the
vaginal vault. Ultrasonography of the pelvis demonstrates a
single endometrial canal containing echogenic material. The
uterus is homogeneous and there is no evidence for an
intrauterine pregnancy. The factor most likely contributing
to this patient's spontaneous abortion is
A.
B.
C.
D.
E.
a bicornuate uterus
a chromosome abnormality of the fetus
hypothyroidism
an incompetent cervix
a submucosal leiomyoma
Explanation:
The correct answer is B. Approximately 50% of early
spontaneous abortions are due to chromosomal abnormalities
of the fetus. Trisomy abnormalities account for 40-50% of
these. The incidence of spontaneous abortions is estimated
at 50% of all pregnancies.
Bicornuate uterus (choice A) is an anatomic abnormality of
the reproductive tract due to the lack of fusion of the
superior portion of the uterine body. This anomaly leads
to difficulty in conception as well as an increased risk
of a spontaneous abortion. This is not the likely cause of
this patient's spontaneous abortion because the ultrasound
did not show two separate endometrial canals.
5
Hypothyroidism (choice C) and other endocrine
abnormalities increase the risk of spontaneous abortion.
This is not the likely cause in this case since the
patient has no prior medical problems. It would be
exceedingly unusual to have hypothyroidism suddenly
present during pregnancy.
An incompetent cervix (choice D) is not the likely cause
of a spontaneous abortion in this patient. The classic
presentation of an incompetent cervix is a sudden
expulsion of a normal sac and fetus between the 18th and
32nd week of pregnancy without antecedent pain or
bleeding.
A submucosal leiomyoma (choice E) is a cause of difficulty
in conception as well as spontaneous abortion. This is not
a likely cause of abortion in this case because the
ultrasound did not demonstrate any focal masses in the
uterus.
A 33-year-old woman comes to the office for a periodic
health maintenance examination. She has no specific
complaints. Her last menstrual period began 10 days
earlier. Physical examination is unremarkable. Pelvic
examination reveals a mobile mass in the left adnexa and an
ultrasound shows that is it is a 4 cm unilocular,
homogeneous, fluid-filled mass. The most appropriate next
step is to
A. aspirate the mass under ultrasound guidance
B. order serum α-fetoprotein and human chorionic
gonadotropin levels
C. order serum CA-125 concentration
D. prescribe oral contraceptive pills
E. repeat the examination in 2-3 months
F. schedule a laparoscopy
Explanation:
The correct answer is E. In premenopausal women, simple
cystic masses are usually benign, functional ovarian cysts
that resolve spontaneously. Functional ovarian cysts are
thought to be formed when an ovarian follicle fails to
rupture during maturation. Reassurance and reexamination
in 2-3 months are recommended.
Since this cyst is most likely a functional ovarian cyst
that will probably resolve spontaneously, it is
inappropriate to aspirate the mass under ultrasound
guidance (choice A). If the mass happens to be malignant
and you aspirate it, this can lead to leakage, peritonitis
and seeding of the peritoneal cavity with the tumor.
Germ cell ovarian tumors are associated with elevated
levels of α-fetoprotein and human chorionic gonadotropin
6
(choice B). This patient most likely has a functional
ovarian cyst rather than a rare, germ cell tumor.
It is unnecessary to order serum CA-125 (choice C) because
this adnexal mass is most likely a functional ovarian
cyst, as opposed to an ovarian malignancy. If the cyst
does not resolve, this test may be indicated in the
future.
Oral contraceptive pills (choice D) are often prescribed
for a functional ovarian cyst that does not resolve
spontaneously. It may be appropriate after reevaluating
this patient in a few months if the cyst is still present.
At this time, a laparoscopy (choice F) for an asymptomatic
simple cyst in a premenopausal woman is too aggressive
since this is most likely benign. This may be the last
resort if the cyst does not resolve spontaneously after a
trial of oral contraceptive pills.
A 28-year-old gravida 3, para 2 woman comes to the clinic
for prenatal care at 11-weeks gestation. Her medical and
surgical history are unremarkable, although she relates a
social history significant for alcohol consumption. She
drinks 1-2 glasses of wine with lunch and 3-4 glasses of
wine with and after dinner on most nights. Given her
history, her fetus is at greatest risk for
A.
B.
C.
D.
E.
a bowel obstruction
a cardiac defect
cleft lip and palate
macrosomia
tall stature
Explanation:
The correct answer is B. Any fetus exposed to alcohol,
especially during the first trimester, is at risk for
fetal alcohol syndrome (FAS). Cardiac malformations,
especially ventricular septal defect (VSD), are noted with
increased frequency among infants exposed to excessive
levels of alcohol in utero. Additional characteristics of
children with FAS may include CNS impairments such as
microcephaly, mental retardation, and attention deficit
disorders. For women with "moderate" alcohol intake,
defined as 1-2 oz of absolute alcohol per day (i.e., 2-4
drinks per day), the risk of FAS is 10%. This woman
consumes 4-6 drinks per day and thus places her fetus at
risk for some component of FAS.
Bowel obstruction (choice A) is incorrect because it is
not found in FAS. Bowel obstruction, which can occur with
duodenal atresia, is found with increased frequency in a
fetus with Down's syndrome, not FAS.
Cleft lip and palate (choice C) is incorrect. Facial
7
dysmorphia found in FAS includes low-set ears, smooth
philtrum, and short palpebral fissures.
Macrosomia (choice D) is incorrect. Growth restriction
(not macrosomia or tall stature) is found either
prenatally or postnatally in FAS.
Tall stature (choice E) is also incorrect. Children with
FAS are small for gestational age and growth restricted.
A 26-year-old African American woman walks into the
emergency department where you are working. She is
37-weeks pregnant and is complaining of a severe
headache for the past 24 hours. She also tells you
that she has noticed that the vision in her right
eye has been extremely blurry since she woke up this
morning. She states that her pregnancy has been
uneventful and that she receives prenatal care at
the hospital clinic. The card she is carrying with
her indicates that all prenatal tests were within
normal limits. Three consecutive blood pressure
readings 15 minutes apart are 156/102 mm Hg, 164/112
mm Hg, and 144/98 mm Hg. The nurse informs you that
her bedside urine dipstick reveals 3+ proteinuria.
On physical examination you find a mild systolic
ejection murmur and 2+ pitting edema of her lower
extremities. A sterile vaginal exam reveals a long
and closed cervix. Tocodynomometer shows irregular
uterine contractions every 8-10 minutes. The
external fetal heart tracing is reassuring. The most
appropriate next step in this patient's management
is
A. an emergent ophthalmology consult
B. intravenous administration of magnesium
sulfate and induction of labor
C. intravenous administration of magnesium
sulfate and a shot of intramuscular terbutaline
to quiet her uterine contractions
D. prolonged external fetal heart monitoring
E. ultrasound examination of the fetus to
evaluate amniotic fluid index and fetal well
being
Explanation:
The correct answer is B. This patient has
preeclampsia; (hypertension that results as a
consequence of pregnancy and regresses postpartum,
proteinuria, and/or pathologic edema (i.e.,
facial/hand)). Preeclampsia may progress to
eclampsia, which is a generalized seizure. Patients
at term (37 weeks and on) who present with
preeclampsia should be induced and given magnesium
8
sulfate at the time of induction as a seizure
prophylaxis.
In some cases patients who are preeclamptic
experience visual changes secondary to retinal
detachment. This patient is complaining of blurred
vision, which may indicate that she needs an
ophthalmologic evaluation (choice A). However, the
top priority is delivery.
This patient is having irregular contractions.
These are normal for any term patient and even if
you were not going to induce her labor, you would
not want to give her a tocolytic such as
terbutaline (choice C).
Fetal evaluation is important in preeclamptic
patients (choice D). In addition, many preeclamptic
patients present with a decreased amniotic fluid
index (oligohydramnios) secondary to placental
insufficiency (choice E). However, delivery is
again the top priority.
A 2-day-old female infant in the neonatal unit has a
distended abdomen and has not passed meconium since birth.
The child was delivered vaginally at term, but her birth
was induced with magnesium sulfate, because the mother was
diagnosed with preeclampsia. The infant's vital signs are:
temperature 38.1 C (100.6 F), blood pressure 70/40 mm Hg,
pulse 130/min, and respirations 22/min. Physical
examination is significant for a distended abdomen. An
abdominal x-ray demonstrates a "bubbly" bowel gas pattern
on the left side of the abdomen. The next step in the
management of this patient is
A.
B.
C.
D.
E.
bowel rest
contrast enema
intravenous fluids
laparotomy
an ultrasonography
Explanation:
The correct answer is B. Clinically, the paucity of
meconium, and the presence of abdominal distension in a
neonate, are suspicious for meconium plug syndrome. The
"bubbly" appearance on the left abdomen reinforces this
consideration. Meconium plug syndrome is more common in
infants after labor induction with magnesium sulfate. A
water-soluble contrast enema will reveal a spindly plug of
meconium in the colon, and often will serve also as a
therapeutic enema. Differential considerations include
small left colon syndrome, which may coexist with meconium
plug syndrome. Also, if therapeutic contrast enema fails
9
to resolve symptoms, Hirschprung's disease or cystic
fibrosis must be considered. Do not confuse these entities
with meconium ileus.
Conservative treatment such as bowel rest (choice A) is
not the appropriate management. A contrast study is
necessary to evaluate for meconium plug syndrome. Failure
to diagnose and treat may result in perforation.
Conservative treatment such as intravenous fluid
resuscitation (choice C) is not appropriate. A contrast
study is necessary to evaluate for meconium plug syndrome.
A laparotomy (choice D) is not indicated for meconium plug
syndrome or small left colon syndrome.
An ultrasonography (choice E) is a useful screening tool
for intussusception, but not for meconium plug syndrome.
A 22-year-old college student comes to the emergency
department with a severe right lower quadrant pain. She
says that the pain started approximately 6 hours ago and
has progressively worsened. She has no significant medical
problems and her only medication is oral contraceptive
pills. She is sexually active with 1 partner, her
boyfriend. Her last menstrual period was 2 weeks ago. Vital
signs are: temperature 37.0 C (98.6 F), blood pressure
120/70 mm Hg, and pulse is 80/min. Abdominal examination is
significant for focal tenderness in the right lower
quadrant. Pelvic examination reveals exquisite tenderness
in the right adnexa, a closed cervical os, and clear
vaginal discharge. Laboratory studies show:
The most likely etiology of this patient's symptoms is
A.
B.
C.
D.
E.
acute appendicitis
diverticulitis
ovarian torsion
a ruptured ectopic pregnancy
a tuboovarian abscess
Explanation:
The correct answer is C. Ovarian torsion is the most
likely cause of abdominal pain localized to the right
adnexa in this patient. She has no significant medical
problems and presents with an acute onset of right lower
quadrant pain in the absence of fever or other laboratory
signs of an infectious process. Ovarian torsion often
occurs in the setting of other ovarian pathology such as a
10
cyst or other lesions in the ovary that causes it to twist
upon its vascular supply. Ovarian torsion is quickly
diagnosed by ultrasonography of the pelvis with the
demonstration of absence of flow to the ovary. This is an
urgent diagnosis and surgery is required to restore blood
flow.
Acute appendicitis (choice A) is not a likely cause of
abdominal pain in this patient. Acute appendicitis is an
infectious process that results from obstruction of the
appendix (usually by a fecalith) and subsequent bacterial
infection. It usually presents with right lower quadrant
pain, fever, and leukocytosis. The presence of peritoneal
signs such as rebound tenderness raises the suspicion for
a ruptured appendix.
Diverticulitis (choice B) is not a likely cause of
abdominal pain in this patient. Diverticulitis is an
infectious process that typically affects older patients
who have diverticulosis. This most often affects the
sigmoid colon, although diverticulosis can occur anywhere
in the colon. Signs and symptoms of diverticulitis include
lower abdominal pain, fever, nausea, vomiting, and
leukocytosis.
A ruptured ectopic pregnancy (choice D) is not a likely
cause of abdominal pain in this patient. This diagnosis
must be considered in any pregnant woman who presents with
abdominal pain, vaginal bleeding, and symptoms of shock.
It is a surgical emergency. This patient has a negative
serum Beta-HCG, which rules out the possibility of an
intrauterine or ectopic pregnancy.
A tuboovarian abscess (choice E) is not a likely cause of
abdominal pain in this patient. A tuboovarian abscess is
an advanced form of pelvic inflammatory disease most often
caused by the spread of bacteria from the lower genital
tract. The most common bacterial pathogens are anaerobic.
Risk factors for pelvic inflammatory disease include those
associated with an increased risk of contracting a
sexually transmitted disease, early age of first sexual
encounter, multiple sexual partners, history of sexually
transmitted disease, and douching. In addition, women
using IUDs are at increased risk for pelvic inflammatory
disease and tuboovarian abscess. Symptoms of tuboovarian
abscess include pelvic pain, fever, leukocytosis, and
abnormal cervical or vaginal discharge in the setting of
pelvic inflammatory disease.
A 46-year-old woman comes to the office for a periodic
health maintenance examination. She is married and has
worked in the local library for 15 years. She states that
she has no medical problems, runs 3 miles every other day,
and feels generally well. She takes a multivitamin, as well
as 1000 mg of calcium every day. She has never smoked
11
cigarettes, and reports drinking wine with dinner when she
goes to a restaurant. Her only surgery was a bilateral
tubal ligation 10 years ago after her third child. When
questioned about her menstrual periods, she said that she
has experienced a 28-30 day cycle for years, with three
days of bleeding. She does mention that for the past three
months her periods have been different, with bleeding about
every 15 days, lasting 2 days. A physical examination and a
pelvic examination reveal no abnormal findings. The most
important next step in this woman's care is to
A. cycle her on a low-dose combination oral
contraceptive pills and give her some extra iron
B. determine FSH and LH levels
C. send her for an ultrasound evaluation of her
pelvic anatomy
D. perform an office endometrial Pipelle biopsy
E. perform a routine Pap smear and send her for a
mammogram
Explanation:
The correct answer is D. With an abnormal bleeding pattern
in a woman over 35, the most important thing to rule out
is endometrial hyperplasia or cancer. The first step in a
workup for endometrial hyperplasia is to perform an easy
office Pipelle biopsy of the endometrium and send it for
pathologic evaluation. If this returns as normal, you can
consider other reasons for her bleeding, which may include
submucosal fibroids, polyps, anovulation, or
perimenopause.
Once you rule out other reasons for her bleeding, such as
endometrial hyperplasia, fibroids, or a polyp, you can
cycle her on OCPs to regulate her bleeding (choice A).
Many perimenopausal women are cycled on low-dose
contraceptive pills as well.
An altered FSH/LH (choice B) may indicate that she is
perimenopausal; however, abnormal bleeding in a patient
her age should be evaluated with a biopsy.
An ultrasound evaluation (choice C) would be reasonable to
evaluate her uterus for any fibroids, and a saline infused
sonogram would show polyps.
Clearly this woman also needs a Pap smear (choice E), but
as a physician the most important thing to do is rule out
cancer with a biopsy. A mammogram is not routinely
recommended by the United States Preventive Services Task
Force until age 50.
A 16-year-old girl comes to the clinic because of a 2-week
history of nausea and vomiting in the morning before
school. The nausea comes on as soon as she wakes up, and is
generally relieved after she "throws her guts up." She
12
attends school and is able to function at her evening job
as a pharmacy clerk. She denies any weight loss. She is
sexually active with "many," different partners and does
not use any form of contraception. She also admits to
injection drug use. She is unsure of the exact date of her
last menstrual period, but thinks that it was about 3-4
weeks ago. Her chart indicates that she has received the
following vaccinations: IPV, MMR, and DaPT. Her blood
pressure is 120/80 mm Hg and pulse is 65/min. Physical
examination is normal. A pregnancy test is positive. You
tell her that she is pregnant and she tells you that she is
going to keep the baby and has no plans to decrease her
sexual activity or drug use. She agrees to undergo HIV
testing in addition to routine tests. Laboratory studies
show:
At this time you should:
A. admit her to the hospital for the treatment of
hyperemesis gravidarum
B. advise her to use oral contraception pills to
prevent future pregnancies
C. have her write a statement about her plans to
continue unsafe sexual activity and drug use and place
it in her chart
D. recommend the hepatitis B vaccine
E. tell her that she must discontinue her drug use if
you are going to continue to treat her
Explanation:
The correct answer is D. This pregnant patient engages in
behavior that puts her at a high risk of contracting
hepatitis B, (and other diseases such as HIV, hepatitis C,
and STDs). Her vaccination history does not include the
hepatitis B vaccine, and her laboratory studies indicate
that she does not have antibodies to the disease. She
should be given the vaccine at this time, even though she
is pregnant, because there have been virtually no adverse
effects on the unborn fetus of mothers who have received
the vaccine during pregnancy. Pregnancy is not a
contraindication to immunization.
Hyperemesis gravidarum (choice A) is a severe form of
nausea and vomiting during pregnancy that affects about 4
in 1,000 women. Individuals with hyperemesis gravidarum
have metabolic abnormalities, dehydration, and weight
loss. Treatment includes intravenous crystalloid fluids,
13
with necessary electrolytes. This patient has typical
"morning sickness," which is nausea and vomiting that
affects the majority of pregnant women.
After she delivers this baby, it may be appropriate to
advise her to use oral contraception pills to prevent
future pregnancies (choice B). However, at this time, she
is pregnant and should not use OCPs. She should, however,
use condoms to reduce the risk of acquiring an infectious
disease.
It is important to document in the patient's chart that
she was advised to stop all high-risk behavior (so the
physician will limit liability if she says that she was
not warned of the affects on the fetus). However, it is
not necessary to have her write a statement about her
plans to continue unsafe sexual activity and drug use, and
place it in her chart (choice C).
Since you want to establish a trustworthy physicianpatient relationship, it would be unadvisable to tell her
that she must discontinue her drug use if you are going to
continue to treat her (choice E). You should try to
explain to her why her behavior might be harmful to her
and her unborn child, but ultimately it is your duty to
treat her, even if you do not approve of her decisions.
A 39-year-old woman comes to the office because of right
lower quadrant and thoracolumbar back pain. She has no
significant past medical history and does not take any
medications. She denies cigarette or alcohol use. A CT scan
of the abdomen and pelvis is obtained and the study is
shown.
14
The preventive measure most likely to have prevented this
outcome is a
A.
B.
C.
D.
E.
chest auscultation
clinical breast exam
clinical thyroid exam
mammography
self breast exam
Explanation:
The correct answer is E. The history of thoracolumbar back
pain and the CT findings of a sclerotic metastasis to the
right hemivertebra are diagnostic for metastatic disease.
The most common cancers for women in this age group are
lung and breast cancer. There is no effective screening
tool for lung cancer. For breast cancer, a self breast
exam picks up most breast cancers before presentation. It
is estimated that more than 80% of cancerous lumps are
15
found by women themselves. While the recommendations are
controversial with many groups having different
guidelines, The American Cancer Society (ACS) guidelines
for the early detection of breast cancer in asymptomatic
women are: (1) Women 30 years of age and older should
perform breast self examination every month. (2) Women,
20-39, should have a physical examination of the breast
every 3 years performed by a health care professional such
as a physician, physician assistant, nurse, or nurse
practitioner. (3) Women 40 and older should have a
physical examination of the breast every year performed by
a health care professional such as a physician, physician
assistant, nurse, or nurse practitioner. (4) Women 40
years of age and older should have a mammogram every year.
A chest auscultation (choice A) rarely detects lung
cancer. Screening chest x-rays have not been shown to be
an effective screening tool.
A clinical breast exam (choice B) is an important part of
screening for breast cancer, but the self breast exam
picks up most cancers.
A clinical thyroid exam (choice C) is important, but
thyroid cancers are rare compared to breast cancer.
A mammography (choice D) is an important part of screening
for breast cancer, but the self breast exam picks up most
cancers. Also, screening mammography is generally not
recommended until age 40.
A 2-day-old male newborn in the neonatal unit has abdominal
distension. The birth was a 39-week normal vaginal
delivery. There is a family history of cystic fibrosis.
Vital signs are: temperature 38.1 C (100.6 F), blood
pressure 74/40 mm Hg, pulse 132/min, and respirations
21/min. On physical examination, the abdomen is distended
and bowel sounds are high pitched. An x-ray of the abdomen
shows multiple dilated loops of both small and large bowel.
The next step in management of this newborn is to
A.
B.
C.
D.
E.
administer oral stool softener
order a barium enema
order a CT scan of the abdomen
perform a sweat test
send CFTR gene analysis
Explanation:
The correct answer is B. This child has clinical and
radiographic evidence of meconium ileus. This occurs when
hard meconium impacts in the ileum. Cystic fibrosis (CF)
is a risk factor for meconium ileus. Indeed, this entity
occurs in adults with CF as well. Barium enema reveals a
microcolon and filling defects in the ileum consistent
16
with meconium plugs. The enema is therapeutic as well. The
differential consideration is ileal atresia; the 2
entities are distinguished by a barium enema.
Oral stool softeners (choice A) may be used in cases of
meconium illness, but mechanical disimpaction with a
contrast enema is necessary.
A CT scan of the abdomen (choice C) is a second-line study
used for problem solving. The first-line modalities in the
newborn with suspected meconium ileus are abdominal x-rays
and contrast enemas.
The sweat test (choice D) along with nasal potential
testing are 2 initial screening tools to detect cystic
fibrosis. These must be confirmed with genotyping. This
child needs immediate workup for his gastrointestinal
problems and that must take priority over the sweat test.
CFTR gene analysis (choice E) is the definitive test to
evaluate for cystic fibrosis. This child needs immediate
workup for his gastrointestinal problems and that must
take priority over gene analysis.
A 17-year-old runaway comes to the emergency department
because of a 24-hour history of lower abdominal pain and
vomiting. She tells you that she hates doctors and
hospitals and is only here because another girl on the
street told her that "this may be serious." She asks you to
give her medicine quickly so she can leave. She is sexually
active with multiple partners and she "occasionally" uses
condoms for contraception. She lives "on the streets" and
begs for money at the doorways of banks. She has not
received any medical care in 6 years. Her last menstrual
period was 9 days ago. She is unsure if she ever had a
sexually transmitted disease in the past. Her temperature
is 38.8 C (101.8 F), blood pressure is 110/70 mm Hg, and
pulse is 65/min. Physical examination shows bilateral lower
abdominal tenderness, but rebound tenderness and guarding
are absent. Pelvic examination shows cervical motion
tenderness, adnexal tenderness, and a yellowish-white
cervical discharge. There are no palpable masses. A urine
pregnancy test is negative. Cervical cultures are taken and
sent to pathology for evaluation. The erythrocyte
sedimentation rate and C-reactive protein level are
elevated. The most appropriate next step is to
A. admit her to the hospital and begin cefoxitin and
doxycycline therapy, intravenously
B. admit her to the hospital and prepare her for an
immediate operation
17
C. do a culdocentesis
D. prescribe ofloxacin and metronidazole therapy,
orally, and discharge her
E. try to contact her male sexual partners for
evaluation and treatment
Explanation:
The correct answer is A. The diagnosis of pelvic
inflammatory disease (PID) is typically based on clinical
findings such as lower abdominal tenderness, cervical
motion tenderness, adnexal tenderness, a vaginal or
cervical discharge, fever >38.3 C, (101.0 F), elevated
erythrocyte sedimentation rate and C-reactive protein, and
laboratory documentation of N. gonorrhoeae or C.
trachomatis. Treatment should be given even before the
culture results return to reduce the risks of infertility
and progressive infection. Admission to the hospital is
important for this patient because compliance and followup cannot be assured since she is homeless and has already
said that she hates doctors and hospitals. There is a
chance that this patient may take the prescription and not
return, so admission is indicated.
It is not necessary to admit her to the hospital and
prepare her for an immediate operation (choice B). She
most likely requires intravenous antibiotic therapy, not
surgery. Surgery may be indicated if there is a suspicion
of appendicitis, a ruptured ovarian cyst, or an ectopic
pregnancy. However, this patient seems to have a classic
case of PID. A tuboovarian abscess (TOA) may also require
surgical evaluation and drainage. Patients with a TOA
often have fever >39.5 C, (103.1F), tachycardia, and
severe pelvic pain.
A culdocentesis (choice C) is not necessary at this time.
This procedure, which is transvaginal sampling of the culde-sac, may be performed to support the diagnosis of an
ectopic pregnancy if clotted blood is found. It may also
be used to obtain bacteriologic samples. It is not
indicated at this time because there is a cervical
discharge present that has been sent to pathology for
evaluation.
If this patient with PID was known to be a compliant
patient and follow-up within 72 hours can be assured, it
is appropriate to prescribe ofloxacin and metronidazole
therapy, orally, and discharge her (choice D). Since she
is homeless and already said that she hates doctors and
hospitals, compliance and follow-up cannot be assured, and
so she should be treated with in-patient therapy.
Admission for all adolescents with PID used to be
recommended, but this is no longer true because there is
not a lot of data to support this recommendation. However,
admission is recommended for the following individuals
including pregnant patients with PID, an abscess or
peritonitis, HIV patients with PID, and all those that
18
fail outpatient therapy.
Evaluating and treating her male sexual partners (choice
E) is important because this condition is due to a
sexually transmitted disease, however, this patient
requires immediate therapy and so contacting sexual
partners should wait until therapy for this patient is
initiated.
You are called to the labor floor in a community hospital
because a 25-year-old patient of yours, who is 37 weeks
pregnant, just presented to the triage area complaining of
painful uterine contractions every 3 minutes lasting 90
seconds. Fetal well-being is assured via external fetal
heart monitoring, and a sterile vaginal exam reveals a
cervix that is 6-cm dilated. Her blood pressure is 90/50 mm
Hg and urine dip is negative. You check her prenatal chart.
She has had an uneventful normal pregnancy course, with
prenatal care starting at 8 weeks. Prenatal labs were
significant only for rubella non-immune and group B
Streptococcus in a urine culture at 28 weeks. In addition
to general intravenous hydration, the most appropriate
management at this time is to administer
A.
B.
C.
D.
E.
ampicillin, intravenously
magnesium sulfate, intravenously
meperidine, intravenously
oxytocin, intravenously
Rubella vaccine
Explanation:
The correct answer is A. This is a full-term patient in a
normal progressive labor pattern with a reassuring fetal
heart tracing. She has risk factors for Group B Strep
infection with a positive urine culture during pregnancy.
The American College of Obstetrics and Gynecology
recommends Group B Strep prophylaxis in labor for patients
with risk factors. The treatment is penicillin or
ampicillin in active labor.
Magnesium sulfate (choice B) is given to pre-eclamptic
patients for seizure prophylaxis. This patient has normal
blood pressure and no proteinuria, and is therefore not
pre-eclamptic.
IV meperidine (choice C) can be used for pain relief. It
is usually used in early labor (prior to 4-cm dilation),
and is not as appropriate as choice B.
IV oxytocin (choice D) is often used for labor
augmentation in laboring patients who are in a
dysfunctional labor pattern. This patient is having
regular contractions and has cervical change. Pitocin is
not necessary at this point.
19
Rubella vaccine (choice E) is given post partum.
A 12-year-old pregnant girl comes to the clinic because of
a 3-day history of a fever and an "itchy rash." Her little
brother had a similar rash 2 weeks earlier. She has
received routine prenatal care and she has had an
uneventful pregnancy so far. Her temperature is 38.3 C
(101.0 F). Physical examination shows a generalized
vesicular rash in various stages of evolution. There are
vesicles and crusted lesions on her arms, legs, trunk, and
face. A complete blood count and liver function tests are
normal. At this time you should
A. administer the varicella vaccine
B. admit her to the hospital for intravenous
acyclovir therapy
C. advise her to stay home from school until all
lesions are dried and crusted
D. recommend salicylates for control of fever
E. tell her that she is no longer contagious since
the rash has already appeared
Explanation:
The correct answer is C. This patient most likely has
chickenpox, which is caused by the varicella-zoster virus.
It is recommended that all children with the chickenpox
stay home from school until all lesions are dried and
crusted. Fetal infection may occur and can result in limb
atrophy, skin scarring, central nervous system
complications, and even death. Some believe that oral
acyclovir should be given to pregnant women with
varicella, but this issue is controversial because the
risks and benefits to the fetus are unknown. The
varicella-zoster immune globulin should be given to
pregnant women without a history of chickenpox provided
that significant exposure to the virus has occurred. It is
not routinely given to pregnant women who have the
infection.
The varicella vaccine (choice A) is a live-attentuated
preparation that is recommended for healthy individuals 12
months or greater who have not had varicella. It is not
given to individuals with the disease.
Admitting her to the hospital for intravenous acyclovir
therapy (choice B) is sometimes used for pregnant patients
with serious complications of varicella. This patient
seems to have an uncomplicated case of varicella (she does
not seem to have complications such as thrombocytopenia,
hepatitis, meningitis, encephalitis, or arthritis).
It is inappropriate to recommend salicylates for control
of fever (choice D) because salicylates have been linked
20
to Reye syndrome when given during a varicella illness.
If she had erythema infectiosum, it would be correct to
tell her that she is no longer contagious since the rash
has already appeared (choice E). However, this patient
most likely has chickenpox and is therefore considered
contagious until all the lesions are dried and crusted.
A 31-year-old woman and her husband have been unable to
conceive for the past 3 years. The woman reports that for
10 years, she has had irregular menses, which occur between
21 to 60 days apart. She states that she is otherwise in
good general health and that she and her husband have been
competing in marathons for over 8 years. Her physical
examination is normal. One year ago, her husband underwent
semen analysis, which was normal. The study that would most
likely be most informative about this patient's infertility
is
A.
B.
C.
D.
E.
basal body temperature record
a CT of the sella turcica
a hysterosalpingogram
laparoscopy
thyroid stimulating hormone (TSH) level
Explanation:
The correct answer is A. A chart recording a woman's basal
body temperature will indicate whether a potential
ovulatory problem may exist. If a biphasic monthly
temperature pattern is found, it confirms luteinization
and ovulation. Female marathon runners are known to
commonly have infrequent or absent menses and ovulations.
Women who have very low body fat and/or participate in
intense exercise are known to have a decrease in
gonadotropin releasing hormone (GnRH), which in turn
results in a diminished release of follicle stimulating
hormone (FSH) and leuteinizing hormone (LH). Ovulatory
dysfunction is responsible for approximately 20-25% of
infertility cases.
A CT of the sella turcica (choice B) is performed as an
initial test to evaluate for pituitary masses. It is not
necessary at this time because the patient's infertility
is most likely due to anovulation related to her long
distance running.
A hysterosalpingogram (choice C) involves an injection of
dye into the uterine cavity in order to evaluate the
contour of the uterine cavity, the patency of the
fallopian tubes, and the ability of the dye to spill
freely into the pelvis. It is useful in the diagnosis of
structural causes of infertility such as congenital
malformations of the uterus, submucosal leiomyomas, and
21
tubal occlusions. It is not necessary at this time because
the patient's infertility is most likely due to
anovulation related to her long distance running.
Laparoscopy (choice D) is not indicated at this time. This
procedure complements information obtained in a
hysterosalpingogram by revealing other tubal
abnormalities, such as small filmy adhesions or peritubal
cysts, that may not be observed on a hysterosalpingogram.
Laparoscopy can also document endometriosis, which has a
strong association with infertility.
Thyroid stimulating hormone (TSH) levels (choice E) are
useful in the evaluation of amenorrhea. It is not
necessary in this patient, since she is most likely
anovulatory secondary to her long distance running.
A 45-year-old man with insulin dependent diabetes mellitus,
peptic ulcer disease, hypercholesterolemia, and a
motorcycle accident 10 weeks prior to this admission, has
been on the telemetry floor for the last 3 days with
shortness of breath and chest pain. His cardiac workup has
been negative. An echocardiogram is pending. Vital signs
have been stable with the exception of nightly low-grade
fevers. There is no previous history of cardiopulmonary
problems. During the accident he sustained multiple
extremity fractures, and a pelvic fracture that required an
external fixator for stabilization. The pin insertion sites
for the external pelvis fixator became infected and the
hardware was removed 10 days prior to this admission. He
has 30 days of intravenous antibiotics remaining. On
morning rounds, he complains of increased left wrist pain
and swelling. He states that when he tries to hold his
coffee cup the wrist pain increases. He denies any left
hand or finger paresthesias. The left distal radius was
fractured in the motorcycle accident. The left arm cast was
removed 1 week ago after non-surgical management. He denies
any new left wrist trauma. Physical examination reveals a
left wrist with diffuse soft tissue swelling, that is
fluctuant dorsally, with mild tenderness to palpation. Mild
erythema and warmth are noted dorsally at the left wrist.
Passive wrist flexion and extension exacerbates the wrist
pain. The neurovascular examination is unremarkable in the
left upper extremity. There is epitrochlear and axillary
lymphadenopathy. The next best step in treating the left
wrist pain, after x-rays are completed, is
A. application of ice to the left wrist and elevation
of the wrist. If the x-rays reveal any abnormalities
then treat accordingly with a cast or splint and
schedule orthopaedic follow up in 1 week
B. aspiration of the fluctuant region of the left
wrist; send the fluid for culture, sensitivity, and stat
22
Gram stain; if the Gram stain is positive then plan for
operative irrigation and debridement of the left wrist
C. aspiration of the fluctuant region of the left
wrist and the wrist joint. If the fluid has any color
component to it, then plan for operative irrigation and
debridement of the left wrist. Send the fluid for
culture and sensitivity, a stat Gram stain, and ensure
broad-spectrum antibiotic coverage
D. if the x-rays are negative, then apply a resting
splint to the left wrist; order occupational therapy to
begin daily left wrist and hand therapy
E. if the x-rays are negative, then aspirate the
fluctuant region of the left wrist; send the fluid for a
Gram stain, culture, and sensitivity; adjust the current
intravenous antibiotic regimen pending the culture
results
Explanation:
The correct answer is C. This patient clinically has a
septic left wrist requiring surgical management. The
aspiration confirms clinical suspicion and allows cultures
to be sent preoperatively. Gram stains are not reliable.
As an example, infected total joint arthroplasties have
positive Gram stains less than 10% of the time. A turbid
aspiration regardless of the Gram stain result would
ensure an operative irrigation and debridement. The
mechanism of the infection is hematogenous. Healing
fractures, such as the left radius, have an increased
blood flow. The infected pin sites at the pelvis resulted
in transient bacteremia and easily seeded the wrist. The
infection required weeks before it had any clinical signs.
. Synovial joint fluid is clear in color and an aspiration
with any color component suggests infection. There are
exceptions to interpretations of aspirate color, but in
this scenario, the infection is quite evident. Clinically,
this patient has a septic joint and the aspiration
confirms the diagnosis. Surgical debridement is the best
management plan.
Apply ice to the left wrist and elevate. If the x-rays
reveal any abnormalitiesy then treat accordingly with a
cast or splint. Schedule an orthopaedic follow-up in 1
week (choice A) is incorrect as this is not a new fracture
or sprain. The infection requires emergent surgical
debridement.
Gram stains (choice B) are unreliable. Surgical decisions
are based upon the history and physical examination.
A warm, tender, fluctuant joint in a patient with this
history and physical exam, has a septic joint until proven
otherwise. Splinting and hand therapy (choice D) are
inadequate and not indicated. Aspiration confirms the
diagnosis. Surgical debridement is the best management
plan.
23
Aspiration and antibiotics (choice E) are not enough. A
fluctuant region has little to no circulation and very
little if any antibiotics will reach this region. Surgery
(i.e., irrigation and debridement) is the only definitive
treatment.
A 25-year-old comes to the emergency department because of
severe right-sided lower abdominal pain for the past 12
hours. She has also experienced fever and chills, but no
change in appetite. She is sexually active with 3 different
partners and they use the "withdrawal" method for
contraception. Her regular menstrual period began 4 days
ago. Her temperature is 39 C (102.2 F), blood pressure is
120/80 mm Hg, pulse is 75/min, and respirations are 20/min.
Physical examination shows right-sided lower abdominal
tenderness with no rebound or guarding. Pelvic examination
shows cervical motion tenderness and a purulent discharge.
A Gram stain of the discharge shows Gram-negative
diplococci within polymorphonuclear leukocytes. A urine
pregnancy test is negative. You prescribe a 14-day course
ofloxacin and metronidazole. She agrees to be compliant
with this therapy and you send her home with a follow-up
visit in 48 hours. She returns to the emergency department
in 12 hours with one of her sexual partners because of
worsening abdominal pain. Her temperature is 39.3 C (102.8
F), blood pressure is 100/60 mm Hg, pulse is 130/min, and
respirations are 28/min. Abdominal examination shows severe
tenderness with guarding and rebound. At this time you
should
A. add cefoxitin to her treatment regimen
B. admit her to the hospital and prepare her for
immediate surgery
C. admit her to the hospital for observation
D. evaluate and treat her sexual partner
E. measure the serum β-subunit of human chorionic
gonadotropin concentration
F. order a CT scan of the pelvis
Explanation:
The correct answer is B. This patient with pelvic
inflammatory disease (PID) most likely has a ruptured
tuboovarian abscess (TOA), peritoneal inflammation and
requires immediate surgery. The abscess needs to be
removed before it leads to more severe peritonitis and
death.
Adding cefoxitin to her treatment regimen (choice A) is
incorrect. In a patient with PID and a TOA you need to
make sure that she has effective anaerobic coverage, which
she does because she was given metronidazole.
24
Admitting her to the hospital for observation (choice C)
is incorrect because as stated above, she needs immediate
surgery because of a TOA.
While it is extremely important to evaluate and treat her
sexual partner (choice D), this patient has a ruptured
TOA, which is a surgical emergency and requires immediate
attention. Evaluating and treating the partner can wait
until the proper arrangements are made for this patient.
It is unnecessary to measure the serum β-subunit of human
chorionic gonadotropin concentration (choice E) because
her regular menstrual period began a few days ago and a
pregnancy test done 12 hours ago was negative.
This patient most likely has a ruptured TOA that requires
immediate surgical attention and ordering a CT scan of the
pelvis (choice F) will only waste important time.
A 59-year-old woman comes to the office for a periodic
health maintenance examination. She and her 55-year-old
sister, who is also a patient of yours, are concerned about
their risk for breast cancer because they have been hearing
so much about it on television. She says, "it seems like
every woman over 40 has it today." The patient's menstrual
period began at age 16 and she had a total abdominal
hysterectomy and bilateral oophorectomy at age 43. She had
one child at age 38. She eats a low fat diet with lots of
fruits and vegetables and does not take hormone replacement
therapy. Her sister's menstrual period began at age 12 and
menopause began at age 53. She had two children at ages 18
and 24. She eats lots of red meat and potato chips and has
been taking hormone replacement therapy since menopause
began. Comparing each detail individually, the factor in
the patient's history that increases her risk for breast
cancer is
A.
B.
C.
D.
E.
age at first pregnancy
age of menarche
age of menopause
diet
lack of hormone replacement therapy (HRT)
Explanation:
The correct answer is A. Since breast cancer is a hormonedependent disease, factors that are associated with
increased hormonal exposure tend to increase the
incidence, while factors that decrease exposure tend to
decrease the incidence. Women who have their first fullterm pregnancy before 18 years old, have a lower risk of
breast cancer than those who have never been pregnant or
those who wait until after they are 35 years old (choice
A).
25
Women who begin menstruating at 12 years of age have a
greater risk than women who began at age 16 (choice B).
Women who go into menopause naturally or surgically before
52 years old, (median age of menopause), have a lower risk
of breast cancer (choice C).
Dietary factors (choice D) are also controversial with
some believing that a high fat intake is associated with a
greater risk of breast cancer. The patient in this case
began menstruating after her sister and began menopause
before her. She eats a low fat diet, and does not take
hormones. She did, however, have a child at an older age
in comparison to her sister.
HRT (choice E) is somewhat controversial, with some
studies finding a small increase in the incidence of
breast cancer with HRT usage.
A 24-year-old woman comes to the clinic for a periodic
health maintenance examination. She has no complaints. She
exercises daily, eats a low fat diet, drinks "a couple of
beers" with friends on the weekends, and is a "social"
cigarette smoker. She has multiple sexual partners and uses
oral contraceptive pills as birth control. She does not use
condoms because "it is not as pleasurable." Her blood
pressure is 110/70 mm Hg and pulse is 60/min. Her physical
examination is unremarkable. You perform a pelvic
examination and send a Pap smear to the laboratory for
evaluation. The results, which return 5 days later, show
two superficial squamous cells with sharply demarcated,
large perinuclear vacuoles and alterations in the chromatin
pattern. They use the term "koilocytic atypia." At this
time the most correct statement about her condition is:
A. Acyclovir will decrease the shedding of the
organism
B. Her Pap smear findings are unrelated to her sexual
activity
C. She and her sexual partners should be treated with
metronidazole
D. She may be at an increased risk for developing
cervical cancer
E. She will develop pelvic inflammatory disease with
perihepatitis
Explanation:
The correct answer is D. This patient most likely has an
infection with the human papilloma virus (HPV) and
depending on the viral subtype (types 16, 18, 31 and 33),
she may be at an increased risk for developing cervical
carcinoma. Subtypes 6 and 11 are associated with condyloma
acuminata which manifests as genital warts. The typical
26
Pap smear findings of a HPV infection include a
perinuclear halo and a wrinkled nucleus. HPV infection is
associated with cervical dysplasia which may progress to
invasive squamous cell carcinoma.
Acyclovir may decrease the shedding of the organism
(choice A) if this patient has a herpes infection. A Pap
smear of a patient with herpes typically shows giant
multinucleated cells with eosinophilic intranuclear
inclusions. This patient most likely has an HPV infection,
not a herpes infection.
Her Pap smear findings are unrelated to her sexual
activity (choice B) is incorrect because she most likely
has an HPV infection, which is a sexually transmitted
disease.
If this patient had a trichomonas infection, she and her
sexual partners should be treated with metronidazole
(choice C). However, she most likely has an HPV infection.
Trichomonas often presents with a frothy vaginal
discharge. A wet mount of the discharge will show motile,
flagellated organisms.
She will develop pelvic inflammatory disease (PID) with
perihepatitis (choice E) is incorrect because this patient
most likely has an infection with HPV, which is associated
with condyloma acuminata, cervical dysplasia, and cervical
carcinoma. PID with perihepatitis (Fitz-Hugh-Curtis
syndrome) is typically associated with chlamydia and
gonorrhea infections. Chlamydia and gonorrhea infections
are associated with cervicitis, which may progress to PID.
They are diagnosed with cervical cultures. Chlamydia is
treated with doxycycline, erythromycin, or azithromycin.
Gonorrhea is treated with ceftriaxone.
A 19-year-old woman who is 16-weeks pregnant comes to the
office for her first prenatal visit. She has a history of 2
prior pregnancies, both of which were terminated by
elective abortions. However, she would like to continue
this pregnancy. Her pregnancy to date has been uneventful.
Upon further questioning, she admits that she has been
smoking “crack cocaine” for the past year on and off. She
denies being addicted and says “I can stop using it
whenever I want". You encourage her to try to stop using
crack cocaine and suggest that she enroll in a drug
treatment program. You try to convince her of the harmful
effects of all drug use during pregnancy. You should tell
her that cocaine increases her risk for
A.
B.
C.
D.
cerebral vein thrombosis
eclampsia
placental abruption
placenta previa
27
E. toxic shock syndrome
Explanation:
The correct answer is C. Cocaine use during pregnancy is
known to be associated with vascular problems due to its
vasoconstricting properties. It is associated with
placental abruption, preterm delivery, intrauterine growth
retardation, and birth defects involving the skull, limbs,
and urogenital system. Placental abruption is the
premature separation of the placenta from the uterine
wall. The greater the amount of separation, the greater
the risk to both mother and fetus. Conditions that
increase the risk of abruption include chronic
hypertension, smoking, toxemia, and the usage of any form
of cocaine. Blunt trauma to the abdomen during pregnancy
can also increase the risk of abruption.
A cerebral vein thrombosis (choice A) and a dural venous
thrombosis are seen in hypercoagulable states such as
pregnancy. There is no increased risk, however, of a
cerebral vein thrombosis with the use of cocaine.
Eclampsia (choice B) is the occurrence of one or more
seizures in association with the syndrome of preeclampsia.
The cause of preeclampsia is not known. Preeclampsia
affects at least 5% of all pregnancies and is a rapidly
progressive condition characterized by hypertension,
edema, and proteinuria. Typically, preeclampsia occurs in
the late second or third trimesters. The use of cocaine
during pregnancy is not associated with an increased risk
of developing preeclampsia or eclampsia.
Placenta previa (choice D) is defined as the implantation
of the placenta over or near the internal os of the
cervix. The exact etiology of placenta previa is unknown.
This abnormal implantation of the placenta is thought to
be related to multiple gestations, advanced maternal age,
previous cesarean delivery, previous abortion, and
possibly smoking. There is no association with cocaine
use.
Toxic shock syndrome (choice E) is caused by a highly
invasive group A Streptococcus with or without necrotizing
fasciitis and is associated with shock and organ failure.
There is no increased risk of toxic shock syndrome during
pregnancy or with cocaine use. A form of toxic shock
syndrome caused by Staphylococcus aureus is a rare illness
occurring mostly in menstruating women who use highabsorbency tampons.
A 32-year-old woman is admitted to the hospital because of
severe left-sided abdominal pain and vaginal bleeding for
the past 24 hours. She says that her last menstrual period
was 7 weeks ago, which is unusual because her menstrual
period "always" occurs every 29 days. She states that she
28
may be pregnant, but also says that she has started a new
job and has been working long hours lately. She just
assumed that her cycle is "adjusting to this new
lifestyle." She is married, does not have any children, and
has never been pregnant. Physical examination shows a
tender left-sided adnexal mass and blood at the cervix. An
ultrasound shows a left-sided adnexal mass. Beta-human
chorionic gonadotropin levels are positive, but low for
gestational age. Her blood type is O, Rh-negative. A
laparoscopy is performed and an ectopic pregnancy is
resected. She recovers from the procedure and is scheduled
to be discharged in 24 hours. The most appropriate next
step in management to
A.
B.
C.
D.
E.
administer betamethasone
administer medroxyprogesterone
administer RhoGAM
infuse type O-negative blood
schedule a dilation and evacuation
Explanation:
The correct answer is C. Although this patient is Rhnegative, RhoGAM is indicated (Rh immunoglobulin) because
there is a risk of isoimmunization. During an ectopic
pregnancy, the woman may be exposed to fetal red blood
cells and develop antibodies to fetal antigens, which can
affect subsequent pregnancies. If the mother is Rhnegative and develops antibodies, and the subsequent fetus
is Rh-positive, the antibodies may cross the placenta and
cause hemolysis of fetal red blood cells. This process can
lead to significant fetal morbidity and mortality,
including fetal anemia and erythroblastosis fetalis.
Isoimmunization can usually be prevented by giving RhoGAM
to any Rh-negative pregnant woman who has an episode of
bleeding. Other times that an RhoGAM is necessary is
following an ectopic pregnancy, a spontaneous or induced
abortion, or an amniocentesis. If there is no bleeding
during the pregnancy, then it should be given routinely at
about 28 weeks and again within three days of delivering a
Rh-positive neonate.
Betamethasone (choice A), which is a steroid, is given to
patients experiencing preterm labor to enhance pulmonary
maturity. An infant born before 32-34 weeks of gestation
is at risk of developing respiratory distress syndrome
(RDS). Treatment with steroids has been shown to reduce
the incidence of RDS and other complications associated
with preterm labor and birth.
An injection of medroxyprogesterone (choice B) is
inappropriate at this time because the patient has not
requested this method of birth control.
Medroxyprogesterone, which is also called Depo-Provera, is
a long-acting injectable hormonal contraception that is
29
typically given every three months. It is not standard
practice to routinely administer medroxyprogesterone
following an ectopic pregnancy.
An infusion of type O-negative blood (choice D) is not
indicated because this patient is stable and does not
require a blood transfusion.
Dilation and evacuation (choice E) is a method of
pregnancy termination in which the cervix is gradually
dilated and the uterine contents are extracted. This
method is used for second trimester therapeutic abortions,
intrauterine fetal death, and incomplete and missed
abortions. Dilation and evacuation is not used to treat an
ectopic pregnancy.
A 38-year-old woman is brought to the emergency department
by ambulance after being found moaning in the middle of the
street near a shopping cart with her belongings. She is
clutching her abdomen, moaning, and rocking back and forth.
She is wearing tattered clothes, is unkempt and disheveled.
After partially undressing her in the emergency department,
she appears to be pregnant with a large gravid abdomen. Her
underwear is stained with approximately 10cc of dark blood.
The nurse finds drug paraphernalia in her jacket pocket.
Her temperature is 37.3 C (99.1 F), blood pressure is
142/94 mm Hg, pulse is 125/min, and respirations are
26/min. On examination, she has pinpoint pupils and a
tetanic (continuous) abdominal contraction. Her fundal
height is 34 centimeters. No pelvic examination is
performed. On brief ultrasonic evaluation, a fetus is
visualized, but no fetal cardiac activity is visualized. A
urine toxicology screen is positive for cocaine. Laboratory
studies show:
The patient's vaginal bleeding is most likely caused by
A.
B.
C.
D.
E.
abruptio placenta
bleeding associated with labor and cervical change
vasa previa
placenta previa
preeclampsia
Explanation:
The correct answer is A. The differential diagnosis of
bleeding in the third trimester of pregnancy includes
30
placental abruption, placenta previa, vasa previa, and
bleeding associated with labor. Abruption refers to
separation of the placenta from the uterus, which leads to
bleeding and sometimes to an intrauterine fetal demise.
This can occur in any pregnancy but is more common in the
setting of drug abuse, smoking, and abdominal trauma
(falling on the abdomen, domestic violence, motor vehicle
accident). The patient is at high risk for an abruption
secondary to her cocaine abuse. Although only 10 cc of
blood is visible externally, it is possible that 1-2
liters of blood is being sequestered behind the placenta.
This would explain her anemia, low platelet count, and her
tetanic contraction (a common uterine response to a severe
abruption).
Bleeding associated with labor is often called "bloody
show" (choice B). It is caused by cervical change and is
typically small volume, bright red, and mixed with
cervical mucus.
A vasa previa refers to implantation of the fetal
umbilical cord into the fetal membranes instead of the
placenta (choice C). This condition is extremely difficult
to detect prior to labor. When rupture of membranes
occurs, the umbilical cord may be severed resulting in
fetal exsanguination. This bleeding is bright red and
occurs very abruptly.
A placenta previa refers to a placenta that covers the
internal cervical os (choice D). Bleeding associated with
a placenta previa can be both of small volume or can be
catastrophic. Placental abruption and previa are the most
common causes for bleeding in the third trimester.
Although it is possible that the patient has a previa, her
cocaine abuse is more consistent with an abruption.
Preeclampsia is not associated with vaginal bleeding
unless an abruption occurs (choice E). The patient's blood
pressure is slightly elevated at 142/94. However, the
patient's recent cocaine use is more likely to be causing
her high blood pressure than preeclampsia. She shows no
evidence of hemoconcentration or elevated creatinine that
can occur with preeclampsia.
A primigravid 24-year-old woman at 34-weeks gestation comes
to the clinic because of a pruritic rash that has been
developing on her abdomen over the last week. She is
otherwise well and has had normal prenatal visits and blood
work. She lives with her husband who does not appear to be
affected. Physical examination shows numerous 1-2 mm
erythematous, edematous vesicular papules along the
periumbilical striae distensae but sparing the umbilicus.
There is an extension of similar lesions to her upper
medial thighs. No pustules, bullae, or burrows are
appreciated. The remainder of the physical examination is
unremarkable. A biochemical profile is normal. She should
31
be told that she has
A. a form of pustular psoriasis and should be started
on systemic corticosteroid
B. Herpes gestationis and will most likely develop
similar lesions in subsequent pregnancies
C. prurigo gravidarum and will be at increased risk
for postpartum hemorrhage
D. pruritic urticarial papules and plaques of
pregnancy and it typically will resolve postpartum
without treatment
E. scabies and needs to be treated with permethrin
Explanation:
The correct answer is D. Pruritic urticarial papules and
plaques of pregnancy (PUPPP) is a relatively common,
intense pruritic dermatosis of pregnancy that typically
occurs in the third trimester in primigravidas. PUPPP
typically develops on the abdomen, especially with
periumbilical striae distensae, while the umbilicus is
usually spared in contrast to patients with Herpes
gestationis. PUPPP is characterized by 1-2 mm erythematous
papules that coalesce to form urticarial plaques that may
involve a large portion of the abdomen. PUPPP usually
responds to topical corticosteroid for symptomatic relief
and remits postpartum. It does not recur in subsequent
pregnancies, flares with ingestion of oral contraceptives,
and is not associated with increased fetal morbidity and
mortality.
Impetigo herpetiformis, a rare form of pustular psoriasis
(choice A) in pregnant females, is marked by acute,
febrile onset with development of grouped erythematous
plaques that are rimmed with small sterile pustules.
Lesional plaques predominate in flexural areas and expand
peripherally. It is usually associated with significant
constitutional symptoms and hypocalcemia with secondary
delirium, seizures, and tetany. It usually promptly remits
after delivery but may recur in subsequent pregnancies.
Herpes gestationis (choice B), also known as pemphigoid
gestationis, is a rare autoimmune disease that occurs
exclusively during the second or third trimester of
pregnancy, or during the immediate postpartum period. It
is characterized by intense pruritic papules and tense
blisters. Classically, Herpes gestationis presents as
urticarial eruption on the abdomen that usually evolves
into a pemphigoid like eruption, sparing the face, mucous
membranes, palms, and soles. The umbilicus is usually
involved, in contrast to PUPPP. The disease tends to recur
during subsequent gestations and one quarter of women
experience flare-ups with use of oral contraceptives.
32
Prurigo gravidarum (choice C) is a hepatic condition that
usually occurs late in pregnancy. The initial cutaneous
manifestation is pruritus, which usually precedes onset of
jaundice by 2-4 weeks. It tends to remit soon after
delivery but typically recurs in subsequent gestations. It
may recur in susceptible individuals after exposure to
oral contraceptives. Some reports suggested increased
incidence of prematurity, stillbirth, and postpartum
hemorrhage.
Scabies (choice E) is infestation with the human itch mite
Sarcoptes scabiei, which is spread primarily by person-toperson contact. The chief symptom is pruritus, which can
occur on any part of the body but most commonly occurs on
the hands, waist, and genital area. The most diagnostic
lesions are burrows, which are small, crooked lines 4-6 mm
in length that are most common in the web spaces of the
fingers, sides of hands, flexor surfaces of wrist, and
lateral and medial surfaces of the foot nearest the heel.
Mainstay of therapy include permethrin 5% and lindane.
Ivermectin single oral dose has been reported to be
effective for severe scabies.
A 19-year-old college student comes to the student health
service because she "desperately needs help." Her
boyfriend is visiting for the weekend and they had
unprotected intercourse that morning. She usually uses
condoms for contraception, but had run out. Other than
exercise-induced asthma, she has no medical problems. The
only medication that she uses is an inhaler for her
asthma. She does not smoke cigarettes or drink alcohol.
She admits to some occasional marijuana use. She saw a
gynecologist 5 months ago and reports a normal Pap smear
at that time. She has normal menstrual periods every
month, and is expecting her menses any day. She wants
emergency contraception in order to prevent pregnancy.
Prior to giving her the appropriate medication, the most
appropriate next step is to
A. obtain a gonorrhea/chlamydia probe from her
endocervix
B. order a beta HCG test
C. order serologic testing for HIV
D. order a urine drug toxicology screen
E. perform a vaginal wet mount
Explanation:
The correct answer is B. Prior to giving a patient
emergency contraception, it is necessary to establish
that she is not clinically pregnant. A beta HCG test is
the appropriate test to make sure she is not already
pregnant.
33
Unprotected intercourse does put a patient at risk for
other sexually transmitted diseases, such as gonorrhea,
chlamydia (choice A), trichomonas, and HIV (choice C). It
is necessary to counsel the patient about this, and offer
her testing. However, this is not a necessity prior to
giving emergency contraception.
A urine toxicology screen (choice D) is not necessary
prior to administration of emergency contraception.
However, a physician may not want to give increased
estrogens to a woman who smokes tobacco, as this
increases chances for thromboembolic events.
A wet mount (choice E) is useful for detecting
trichomonas. It is not necessary to diagnose and treat
this before providing emergency contraception.
You get a call from a pregnant hepatitis B surface antigen
positive woman that you have been taking care of. She is
frantic because she unexpectedly went into labor and
delivered the baby in her bathtub 30 minutes earlier. She
had her husband cut the cord with a clean kitchen knife and
the baby appears to be doing well. She wants to know if you
can see her immediately. She is concerned about how her
hepatitis status affects breast-feeding. You tell her to
bring the baby over to the office and
A. not to breast-feed until the baby received the
proper immunizations
B. that she can begin to breast-feed immediately if
the baby is hungry
C. that her newborn will need to be treated with
alpha interferon
D. that her breast milk does not contain hepatitis B
surface antigen
E. that she should not breast-feed the newborn
because of her hepatitis status
Explanation:
The correct answer is B. Even though the breast milk from
a hepatitis B surface antigen positive woman contains the
hepatitis B surface antigen, according to the American
Academy of Pediatrics and the Report of the Committee on
Infectious Diseases, breast-feeding does not significantly
increase the risk of infection. Newborns born to HBsAg
positive women should be given the hepatitis B vaccine and
hepatitis B immune globulin as soon after birth as
possible, but breast-feeding does not need to be delayed
until after immunizations. Therefore, she should be
advised that she can begin to breast-feed immediately if
the baby is hungry.
As stated above, it is not necessary to wait to breast-
34
feed until after the baby is immunized. Therefore, telling
her not to breast-feed until after the baby receives the
proper immunizations is incorrect (choice A).
Alpha interferon (choice C) is used to induce remission in
patients with chronic liver disease. It is not routinely
given to newborns born to HBsAg positive women.
It is incorrect to tell her that her breast milk does not
contain hepatitis B surface antigen (choice D) because
these antigens have been detected in the breast milk of
HBsAg positive women.
It is incorrect to tell her that she should not breastfeed the newborn because of her hepatitis status (choice
E) because as stated above, HBsAg positivity is not a
contraindication for breast-feeding.
A 23-year-old G1P0 is in the hospital after the delivery of
a healthy baby girl 24 hours ago. She had an unassisted
vaginal delivery after a prolonged induction of labor at
41-weeks gestational age. The placenta was expelled 10
minutes after delivery and it appeared to be intact. On the
morning of the second hospitalization day, the patient
reports heavy vaginal bleeding and minimal pain at the
midline episiotomy site. Vital signs are: temperature 37.2
C (99.0 F), blood pressure 136/70 mm Hg, and pulse 90/min.
Bimanual examination of the pelvis reveals a boggy uterus.
The most appropriate initial management of this patient is
A.
B.
C.
D.
E.
hypogastric artery ligation
selective arterial embolization
uterine artery ligation
uterine massage
uterine packing
Explanation:
The correct answer is D. Uterine atony is the most likely
etiology of this patient's postpartum bleeding. The
likelihood of uterine atony increases with prolonged
labor, difficult delivery, and prolonged pregnancy.
Initial management of the patient should include a
thorough pelvic examination for lacerations and signs of
atony, such as a boggy uterus. Uterine massage should be
initiated immediately, if atony is suspected.
Occasionally, appropriate uterine contractions are
stimulated with uterine massage and oxytocics alone.
Hypogastric artery ligation (choice A) is not the
appropriate initial management of this patient. Surgical
measures are reserved for those patients who do not
respond to medical therapy first.
Selective arterial embolization (choice B) is an
interventional radiology procedure which involves an
35
angiogram to localize the specific vessel or vessels that
are bleeding. Metallic coils or other materials are then
used to embolize these selective vessels. It is not the
appropriate initial management of this patient. Invasive
procedures are reserved for those patients who do not
respond to medical therapy first.
Uterine artery ligation (choice C) is not the appropriate
initial management of this patient. Surgical measures are
reserved for those patients who do not respond to medical
therapy first.
Uterine packing (choice E) is not the most appropriate
initial management of this patient. If uterine massage and
oxytocics do not adequately stimulate uterine
contractions, packing the uterine cavity with gauze can be
performed. This is a temporizing measure while the patient
waits for more definitive therapy.
A 32-year-old woman comes to the emergency department with
vaginal bleeding and severe bilateral lower quadrant pain.
She tells you that she recently found out that she is
pregnant and says, “this is my first pregnancy and I'm
afraid I'm going to lose it!” She is pale and in obvious
pain. Her last menstrual period was approximately 8 weeks
ago and she has had irregular prenatal care. Her past
medical history is significant for depression, a treated
chlamydia infection, and migraine headaches. Vital signs
are: temperature 37.0 C (98.6 F), blood pressure 140/90 mm
Hg, and pulse 90/min. Pelvic examination reveals dark blood
in the vaginal vault and a closed cervical os. A urine
pregnancy test is positive and a serum Beta-HCG is pending.
The most appropriate next step in management is to
A. administer methotrexate therapy
B. arrange outpatient follow up appointment with an
obstetrician for better prenatal care
C. order a CT scan of the pelvis
D. order an MRI of the pelvis
E. order ultrasonography of the pelvis
Explanation:
The correct answer is E. The most emergent diagnosis of
concern in a pregnant female presenting with pain and
vaginal bleeding is an ectopic pregnancy. In particular, a
ruptured ectopic pregnancy is a life-threatening condition
and requires emergent surgery. The diagnostic study of
choice is ultrasonography of the pelvis. Ultrasonography
is a specific and quick way to evaluate for an
intrauterine pregnancy. The demonstration of a normal
intrauterine pregnancy dramatically decreases the
likelihood of a concomitant ectopic pregnancy. In fact,
36
the chance of having both occur simultaneously is
approximately 1 in 30,000. Sonographic findings that are
highly suspicious for ectopic pregnancy include the
absence of an intrauterine pregnancy, an adnexal mass, and
free fluid in the pelvis. Ectopic pregnancy is also more
common in women with a history of pelvic inflammatory
disease.
Methotrexate therapy (choice A) is a medical treatment for
ectopic pregnancies. In the past, surgery was the only
treatment available for ectopic pregnancies, even if the
patient was hemodynamically stable. If the ectopic
pregnancy is small and the hormone levels aren't high,
there is a high probability that medical management with
methotrexate will be sufficient. Although this is a
relatively new treatment (it has been used for about 7-8
years), it is one, which has been found to be quite safe
and an alternative to surgery in appropriate cases. There
are side effects associated with methotrexate and it
should not be administered until there is documentation of
an ectopic pregnancy by ultrasonography.
Arranging an outpatient follow up appointment with an
obstetrician (choice B) addresses the patient's needs for
better prenatal care. It does not, however, address the
need to diagnose a potentially life-threatening condition
such as a ruptured ectopic pregnancy.
A CT of the pelvis (choice C) is not an appropriate step
in the management of a pregnant patient. CT of the body,
and particularly of the pelvis, exposes the developing
fetus to harmful radiation. Ultrasonography of the pelvis
is more specific and sensitive than a CT in the evaluation
of a pregnancy without the danger of radiation exposure.
MRI of the pelvis (choice D) is not an appropriate step in
the management of this patient. Ultrasonography is the
diagnostic tool of choice in the evaluation of
intrauterine and potentially ectopic pregnancies. MRI of
the pelvis is useful to evaluate masses such ovarian
lesions and leiomyomas of the uterus. There is not a lot
of information regarding the effects of MRI on the
developing fetus at this time.
A 48-year-old homemaker comes to the office for a periodic
health maintenance examination. Although this is the first
time she has seen a doctor in several years, she is in good
health with a surgical history remarkable for an
appendectomy at age15. She has been married for 18 years
and has 2 children. She goes to aerobics class 3 times a
week, drinks approximately 2 glasses of wine a night, and
has smoked about 5 cigarettes a day for 28 years. She still
gets regular periods and has no signs or symptoms of
menopause. She does state that in the past few weeks she
has noted some blood on the toilet paper when she uses the
37
bathroom after intercourse with her husband. On pelvic exam
you find a 1.5 cm friable lesion, which appears to be
protruding from her internal os. In addition, you suspect a
Candida infection from gross inspection. The most
appropriate next step is to
A. arrange for cervical cone biopsy
B. do colposcopy
C. perform a routine Pap test and await pathology
results before proceeding
D. take a biopsy of the lesion
E. treat the yeast infection and ask her to return
for reexamination in 1 week
Explanation:
The correct answer is D. A biopsy should always be taken
immediately when a cervical lesion is found on physical
exam. With the widespread use of the Pap test, most cases
of postcoital bleeding are due to benign causes such as
cervicitis, cervical eversion, cervical polyps, or
prolapsed submucous fibroids. However, cervical cancer
must be ruled out. Once cervical cancers grow to
sufficient size they often cause postcoital or
intermenstrual bleeding and sometimes cause significant
hemorrhage.
Cervical cone biopsy (choice A) is not indicated at this
point, as the diagnosis can most likely be made with less
invasive means.
A colposcopy (choice B) can sometimes be helpful in
determining the appropriate area to biopsy. However, when
a gross lesion presents, a colposcopy is not necessary to
aid in biopsy.
A routine Pap test (choice C) can be performed, but should
not be relied upon as the sole diagnostic procedure.
Sensitivity of the Pap test in detecting invasive cervical
cancer can be as low as 50%. As stated above, a biopsy
should be obtained immediately and you should not await
the results of the Pap smear.
This patient's yeast infection should be treated (choice
E), but not at the expense of taking a biopsy. The yeast
infection will not affect the biopsy results.
A 27-year-old woman who is 3 weeks postpartum, comes to the
clinic because of left breast pain, body aches, and fever.
She had a normal vaginal delivery and an uneventful
postpartum period. Four days ago, she started feeling
feverish with diffuse body aches. She is concerned, because
she is breast-feeding her infant and her left breast has
also been tender for the last 4 days. She denies cough,
sore throat, or rhinorrhea. Vital signs are: temperature
39.3 C (102.7 F), blood pressure 120/65 mm Hg, and pulse
38
90/min. Breast examination reveals a moderately
erythematous left breast with diffuse tenderness. The right
breast is normal. The rest of the physical examination is
normal. The most appropriate management of this patient is
to
A. advise the patient to discontinue breast-feeding
from the left breast
B. culture her breast milk
C. order an ultrasound of the left breast
D. prescribe oral penicillinase-resistant antibiotic
E. refer her for a surgical consult for abscess
drainage
Explanation:
The correct answer is D. The symptoms of fever, breast
erythema and tenderness, and general body aches in the
postpartum period is most consistent with mastitis. This
breast infection occurs most commonly several weeks after
delivery. The infection is usually unilateral and is
thought to be transmitted from the infant's pharynx.
Staphylococcus aureus is the most common organism, and
therefore, the recommended treatment is a penicillinaseresistant antibiotic such as dicloxacillin.
It is not necessary for the mother to discontinue breastfeeding from the left breast (choice A) in the setting of
mastitis. The route of infection is thought to originate
in the infant's pharynx and there should be no concern of
transmitting the infection to the infant.
Culturing the breast milk (choice B) is no longer
routinely done. Prior studies of breast milk cultures have
demonstrated the prevalence of Staphylococcus aureus in
50% of cases. No other single predominant organism was
isolated in the other 50%. Based on this data, it is
appropriate to treat mastitis with penicillinase-resistant
antibiotics without a culture of the breast milk.
Ordering an ultrasound of the breast (choice C) is not
necessary. The diagnosis of mastitis is made clinically.
A surgical consult (choice E) is not the appropriate
management of mastitis. This patient is clearly presenting
with signs and symptoms of mastitis and there is no
clinical indication of a breast abscess.
A 36-year-old woman comes to the office for evaluation of
abnormal discharge from her right eye. She had seen you for
her annual health maintenance examinations and had a Pap
smear done just 2 weeks prior. After diagnosing
conjunctivitis and giving her the appropriate treatment,
you look to see if her Pap smear results are back yet. You
notice that the smear had no cytologic abnormalities, but
that there were no endocervical cells present on the smear.
39
She is a non-smoker, has yearly Pap tests that have never
been abnormal, and has been in a monogamous relationship
for 12 years. The most appropriate next step is to
A. advise her that she needs increased monitoring,
and that you will perform a repeat Pap smear in 6 months
B. advise her that since she is at very low risk for
an abnormality, she can wait until her annual exam next
year for a re-sample of her endocervix
C. explain to her that although the reading of the
exocervix is valid, you need to re-sample her endocervix
D. explain to her that she needs to repeat the Pap
test, as these results are not valid
E. perform a colposcopic examination of her cervix
Explanation:
The correct answer is B. The presence of endocervical
cells on a Pap test is regarded as evidence of adequate
sampling of the transformation zone during cytologic
screening of the cervix. When these cells are absent, it
indicates that this area may not have been sampled. This
is considered a satisfactory, but limited smear. In
patients with no known risk factors (i.e., prior abnormal
Pap test, multiple sexual partners, smoking) the American
College of Obstetricians and Gynecologists recommends that
the physician may defer to repeating the Pap test in 12
months.
It is inappropriate to tell her that she needs increased
monitoring (choice A) because these results do not
indicate any abnormality. It is simply sampling error.
If this were a high risk patient (i.e., prior abnormal Pap
test, multiple sexual partners, smoking), a re-sampling
only of the endocervix (choice C) only would be the
appropriate next step. However, since she is not at high
risk, you can wait for 1 year.
It is inappropriate to tell her that the results of this
test are not valid (choice D) because they are valid, just
not complete.
A colposcopic examination of the cervix (choice E) would
be appropriate if this patient had abnormal cells on her
smear indicating a pre-cancerous lesion.
A 3-day-old male infant in the neonatal unit has bilious
vomiting for 24 hours. The child is inconsolable and will
not feed. Vital signs are: temperature 38 C (100.4 F),
pulse 110/min, blood pressure 80/50 mmHg, and respirations
20/min. Abdominal examination is unremarkable. A barium
enema demonstrates the cecum to be in the left upper
quadrant. There is no right lower quadrant mass on
abdominal x-ray. Intravenous antibiotics and Ringer's
solution are administered. The next step in treating this
40
patient is
A.
B.
C.
D.
E.
bowel rest
intussusception reduction with air
intussusception reduction with contrast
laparotomy
repeat barium enema in 24 hours
Explanation:
The correct answer is D. Bilious vomiting in an infant
means that there is a malrotation with volvulus until
proven otherwise. The radiographic findings of the cecum
in the left upper quadrant confirms this clinical
diagnosis. An upper gastrointestinal series would likely
show a bird-beak deformity of the midgut where there is
volvulus of the gut around a mesenteric "Ladd" band. About
20% of malrotation with volvulus is associated with
duodenal atresia, annular pancreas, or a duodenal
diaphragm. About 33% of cases present in the first week of
life and 85% present in the first year of life. The
hallmark of malrotation is ischemia of the midgut as the
superior mesenteric vein and the superior mesenteric
artery are occluded by the twisting mesentery. Immediate
surgery is necessary to prevent death or the loss of much
of the bowel.
Bowel rest (choice A) would lead to high morbidity and
possibly mortality in this patient with midgut malrotation
with volvulus. Untreated midgut malrotation with volvulus
will lead to mesenteric ischemia.
The diagnosis of intussusception is made with either
ultrasound or an enema using either air or contrast medium
(choice B and C). This patient's presentation of bilious
vomiting favors the diagnosis of malrotation with volvulus
rather than intussusception. Intussusception typically
presents in infants from 4 months to 2 years of age, with
alternating lethargy and irritability, colicky abdominal
pain, and currant jelly stools. In addition, there is
usually a right lower quadrant mass on abdominal x-ray.
Repeating the barium enema in 24 hours (choice E) is not
appropriate as this will only delay definitive surgical
treatment. Repeating this study will not contribute to
this patient's management.
A 31-year-old woman comes to the emergency department with
midline abdominal pain that she reports is 10 out of 10 in
severity on a pain scale, with 10 being the worst pain she
has ever felt. The pain has been present for 1 hour. She
has no past medical history and her last menstrual period
was 1 day ago. Her only medication is oral contraceptive
pills. She has no allergies. She had an uncomplicated
pregnancy 3 years ago and had an uneventful normal vaginal
41
delivery. Vital signs are: temperature 37 C (98.6 F), pulse
90/min, blood pressure 100/70 mm Hg, and respirations
15/min. Oxygen saturation is 96% on room air. Physical
examination reveals a woman in obvious pain. There is left
adnexal and midline pelvic pain on palpation. A urine
pregnancy test is negative. Ultrasonography of the pelvis
demonstrates an enlarged left ovary with decreased blood
flow. The most likely complication of this finding is
A.
B.
C.
D.
E.
fibroid degeneration
infertility
ovarian cancer
ovarian cysts
pelvic inflammatory disease
Explanation:
The correct answer is B. A young woman with acute pelvic
pain localizing to one side is suspicious for ovarian
torsion. Associated symptoms may include nausea and
vomiting. The diagnosis was confirmed with endovaginal
ultrasound demonstrating an edematous ovary with decreased
venous and possibly, arterial flow. Complications of
ovarian torsion include infection, peritonitis, sepsis,
adhesions, chronic pelvic pain, and infertility due to the
loss of the viability of the torsed ovary.
Fibroids (choice A) may degenerate over time as they
outgrow their blood supply. Moreover, adventitial fibroids
may be pedunculated and torse on their stalk. Pain from a
degenerating fibroid is rare. Degenerating fibroids have
no association with ovarian torsion.
Ovarian cancer (choice C) is not a complication of
torsion. The typical presentation of ovarian cancer is
diffuse carcinomatosis and abdominal mass or bowel
obstruction. Ovarian cancers can enlarge the ovary and may
predispose it to twisting on its vascular supply.
Ovarian cysts (choice D) are not a complication of
torsion. Benign functional ovarian cysts can get quite
large and may predispose a patient to torsion by causing
the ovary to twist on its vascular supply.
Pelvic inflammatory disease (choice E) is not a
complication of torsion. It is caused by organisms
ascending to the upper female genital tract from the
vagina and cervix. It most commonly is associated with
Chlamydia trachomatis and Neisseria gonorrhoeae, but other
organisms, and in many cases multiple organisms, have been
isolated.
A 27-year-old female police officer comes into the
emergency department where you are working. She is
complaining of a 2-day history of nausea, and vomiting and
some mild left lower quadrant pain. Her vital signs are
42
stable. Physical examination shows mild tenderness to
palpation in the left lower quadrant. Prior to performing
the pelvic examination, the patient informs you that she is
menstruating. The examination is significant only for blood
in the vault. Routine laboratory studies are sent and a
beta hCG returns as 2700. You order a pelvic ultrasound,
which identifies nothing in the uterus. However, there is a
fetal pole in the left fallopian tube. You inform the
patient that she has an ectopic pregnancy and discuss the
option of surgery versus methotrexate therapy. The patient
decides that she would like to try methotrexate.
Appropriate follow-up care for this patient will include
A. another dose of methotrexate on day 4 of therapy
if blood levels of hCG have not fallen by 15%
B. checking beta HCG on day 3 and 7 of methotrexate
therapy
C. immediate laparotomy if methotrexate therapy is
found to fail by day 7 of treatment
D. repeating beta hCG 2 weeks after diagnosis if beta
hCG is found to have fallen greater than 15% from day 4
to day 7 of treatment
E. repeating pelvic ultrasound in 1 week
Explanation:
The correct answer is D. Ectopic pregnancy can be managed
medically in specific situations dependent on size of the
mass and whether or not it has ruptured. Treatment with
methotrexate is calculated according to body surface area.
Patients are counseled extensively on the side effects of
methotrexate and given strict ectopic precautions for
cases of rupture. Beta hCG is routinely drawn on day 4 and
day 7 of treatment. If the beta hCG drops 15% from day 4
to day 7, treatment is thought to be successful and the
patient should return weekly for beta hCG blood draws
until it is negative.
Beta hCG peaks on day 4 of treatment. It is on day 7 of
treatment that another dose of methotrexate may be given
if levels do not fall 15% from day 4 of treatment. Another
dose of methotrexate on day 4 of therapy if blood levels
of hCG have not fallen by 15% (choice A) is incorrect.
Levels are routinely checked on day 4 and day 7 of
treatment, not 3 and 7 (choice B).
If methotrexate therapy is found to have failed by day 7
of therapy another dose of methotrexate can be tried.
Immediate laparotomy (choice C) is indicated in cases of
rupture.
Pelvic ultrasound (choice E) does not need to be repeated
unless indicated by a change in the patient's clinical
status.
43
A 26-year-old woman comes to the clinic because of a 3-day
history of a "bad smelling", grayish colored vaginal
discharge. She says that she has had 4 different sexual
partners in the past 5 years, and she has never had a
sexually transmitted disease. Her current sexual partner is
asymptomatic. Physical examination is unremarkable. Pelvic
examination shows a moderate amount of gray vaginal
discharge at the introitus and adherent to the vaginal
walls. The pH of the discharge is 5.1. There is a fishy
odor released when you expose the discharge to KOH. You
decide to do a wet-mount preparation of the discharge.
Based on the history and pelvic exam, you expect to see
A. epithelial cells with smudged borders
B. Gram-negative intracellular diplococci
C. motile flagellated organisms
D. no cells; a culture would be required for
diagnosis
E. yeast and pseudohyphae
Explanation:
The correct answer is A. Epithelial cells with smudged
borders are often called "clue" cells and are due to
bacteria adherent to the cell membrane which is
characteristic of bacterial vaginosis (BV). BV is a
syndrome seen in sexually active females which may be
asymptomatic but often presents with a thin, white-gray,
foul smelling discharge which people often describe as
"fishy". BV is usually not associated with pruritus,
dysuria, or abdominal pain. BV is the most prevalent
vaginal infection in sexually active females. Although not
completely clear, causes are thought to include
Gardnerella vaginalis, Mycoplasma hominis, and anaerobic
bacteria. In order to diagnose BV, a woman must have at
least 3 of the following symptoms, including a white-gray
non-inflammatory vaginal discharge that adheres to the
vaginal wall, vaginal fluid pH greater than 4.5, a "fishy"
odor to the vaginal fluid either before or after mixing
with 10% potassium hydroxide, and the presence of clue
cells. Metronidazole is the treatment. Treatment of the
sexual partner is not routinely recommended.
Gram-negative intracellular diplococci (choice B) are
indicative of a gonorrheal infection and would be seen on
a Gram stain, not a wet mount, which is prepared with
saline. The infection may be asymptomatic, however it may
be associated with a malodorous, purulent (yellowishgreen) vaginal discharge. A culture on a Thayer-Martin
agar plate is often used to establish the diagnosis.
Enzyme immunoassays are also available. The treatment
includes ceftriaxone and doxycycline (because of the high
incidence of co-infection with chlamydia). Treatment of
the sexual partner is routinely recommended.
44
Motile flagellated organisms (choice C) are seen in
Trichomonas vaginalis. Although the infection is often
asymptomatic, men may develop urethritis, and women may
complain of a frothy vaginal discharge that is green or
yellow in color and may have a musky odor. Women may also
have some lower abdominal tenderness with more severe
symptoms just before or just after menstruation. On exam,
the vaginal mucosa may be erythematous with an inflamed
and friable cervix. Women may also have "strawberry
cervix," a term used when there are multiple petechiae on
the cervix. A wet prep will often show trichomonads with
their flagella and jerky motility. Metronidazole is the
treatment. Simultaneous treatment of the partner is
necessary.
BV can be diagnosed by finding a white-gray noninflammatory vaginal discharge that adheres to the vaginal
wall, a pH > 4.5, an amine odor when combined with KOH,
and clue cells on a wet mount. A culture is not necessary
to establish the diagnosis (choice D).
In a woman with a yeast infection, you would expect to see
yeast and pseudohyphae (choice E) on a wet mount. These
women usually present with complaints of intense pruritus
and burning accompanied by a thick white vaginal
discharge. Yeast infections are caused by overgrowth of
Candida albicans often due to factors such as pregnancy,
antibiotic use, diabetes, and oral contraceptive use. Some
women report predisposition to yeast infections
immediately preceding menstruation. The treatment includes
imidazole creams or suppositories. Treatment of the
asymptomatic sexual partner is not routinely recommended.
A 31-year-old woman comes to the clinic because of "not
having a period for over a year, white discharge from both
nipples, and severe frontal headaches of 2 years duration".
Evaluation reveals a follicle-stimulating hormone (FSH)
level of 6.0 mIU/mL, basal serum prolactin level of 82
ng/mL, and thyroid-stimulating hormone (TSH) of 19 microU/mL (normal ranges: FSH 2-20 mIU/mL, prolactin, <20 ng/mL,
TSH, 0.5-5.0 microU/mL). An MRI reveals pituitary
enlargement with a mass measuring 13 mm in diameter. The
next step in the management of this patient is
A.
B.
C.
D.
E.
evaluation of other pituitary hormones
formal visual field testing
referral to a neurosurgeon
therapy with bromocriptine
therapy with levothyroxine
Explanation:
The correct answer is E. Prolactin, unlike other anterior
45
pituitary hormones, is primarily under tonic inhibition by
dopamine secreted from the hypothalamus. Prolactin release
can be stimulated by many factors including thyrotropinreleasing hormone (TRH). Because TRH is increased in
primary hypothyroidism, the circulating TSH (not TRH)
should be measured in all women who have amenorrhea or
galactorrhea, or both, to exclude hypothyroidism.
Increased TSH levels should be presumed to result from
primary hypothyroidism. Many reports have documented that
primary hypothyroidism can mimic a pituitary tumor and can
lead to profound pituitary enlargement, because of the
hypertrophy of thyrotrophs. After treatment with
levothyroxine, the enlargement of the pituitary should
subside and the patient should become euthyroid.
Evaluation of other pituitary hormones (choice A) is not
indicated at this time. Evaluation of other pituitary
hormones not measured initially such as LH, and GH is not
necessary since the elevation of TSH is diagnostic for
hypothyroidism.
Formal visual field testing (choice B)
this time. Formal visual field testing
if you suspected bitemporal hemianopia
tumor in the area of the sella turcica
the optic chiasm).
is not indicated at
would be important
(caused by a large
putting pressure on
Referral to a neurosurgeon (choice C) is not appropriate
at this time. You should refer her to a neurosurgeon only
after establishing that a tumor, or a macroadenoma, is the
cause of enlargement in the pituitary, and you have
treated the patient with levothyroxine without improvement
in the size of the pituitary and functional status of the
thyroid.
Therapy with bromocriptine (choice D) is incorrect. Since
bromocriptine is a dopamine receptor agonist, it would
decrease prolactin. This may be useful in
hyperprolactinemic states, but not in hypothyroidism.
You are called to see a patient on the postpartum floor of
a small community hospital. She is a 24-year-old woman who
is 2 days postpartum after a normal spontaneous vaginal
delivery of a healthy baby boy. This is her first child.
She has no medical problems, had a pregnancy complicated
only by some early nausea and vomiting, and has never had
surgery before. She was compliant with taking prenatal
vitamins and iron throughout pregnancy and has continued
taking them since her delivery. She has been trying to
breastfeed her son, as she understands that it is
beneficial to bonding and to the overall health of the
46
newborn. She has good milk letdown. Her temperature is 37.2
C (99.0 F) and she is complaining of pain in both breasts.
The only remarkable finding on physical examination is
firm, tender breasts. The most appropriate management at
this time is to
A. advise her to bottle feed only until the fever
subsides without using the breast pump
B. advise her to feed more frequently in addition to
pumping the breasts as needed
C. advise her to use breast pump only and use this
milk for baby feeds
D. apply cold compresses to breasts while continuing
to breast feed
E. begin dicloxacillin therapy while continuing to
breast feed
Explanation:
The correct answer is B. Breast engorgement is a common
problem found in postpartum women. It is characterized by
slight fever and bilateral rock-hard breasts. Clogged milk
ducts can present as localized tender masses. If the woman
is breast feeding, she should be advised to empty her
breasts more often, and is thus encouraged to either pump
or breastfeed with increased frequency. Warm compresses
are also advisable to enhance milk letdown. If the woman
is not breast feeding, she should use cold compresses in
addition to a breast binder.
As explained above, emptying the breasts is what decreases
the chances of engorgement. Thus, it is imperative that
the woman continues to breastfeed or use the breast pump.
Therefore, advise her to bottle feed only until the fever
subsides without using the breast pump (choice A) is
incorrect.
There is no reason that the patient cannot have the baby
latch on. Even in the scenario of sore nipples,
breastfeeding with latch-on is advocated. She should not
be advised to only use the breast pump (choice C).
As explained above, warm compresses facilitate milk
letdown, whereas cold compresses (choice D) are utilized
for women who are not breast feeding.
The main differential diagnosis for engorgement is
mastitis, which occurs in approximately 2% of
breastfeeding women, most commonly occurring between the
first and fifth weeks postpartum. It is manifested by a
sore, reddened area on the breast. The patient will feel
ill, with high fevers, chills, and malaise. Approximately
40% of cases are caused by Staphylococcus aureus and the
first choice antibiotic is dicloxacillin (choice E). The
patient is encouraged to breast feed while being treated.
47
An 18-year-old girl comes to the emergency department with
her mother because of a 12-hour history of lower abdominal
pain and nausea. She is sexually active with 3 different
partners and she "usually" uses condoms for contraception.
She is unsure of the exact date of her last menstrual
period. She has never had a sexually transmitted disease in
the past. Her temperature is 38.3 C (101.0 F), blood
pressure is 110/70 mm Hg, and pulse is 65/min. Physical
examination shows bilateral lower abdominal tenderness, but
rebound tenderness and guarding are absent. Pelvic
examination shows mild cervical motion tenderness and
adnexal tenderness. A small amount of cervical discharge is
present. There are no palpable masses. Cervical cultures
are taken and sent to pathology for evaluation. The most
appropriate next step is to
A. admit her to the hospital and begin cefoxitin and
doxycycline therapy, intravenously
B. do a culdocentesis
C. do urinary B- human chorionic gonadotropin testing
D. prescribe ofloxacin and metronidazole therapy,
orally, and discharge her
E. try to contact her male sexual partners for
evaluation and treatment
Explanation:
The correct answer is C. It is likely that this patient
has either an ectopic pregnancy or pelvic inflammatory
disease. The cervical motion tenderness and bilateral pain
may be more consistent with PID, which is often
characterized by those symptoms and adnexal tenderness, a
vaginal or cervical discharge, a fever >38.3 C, (101.0 F),
elevated erythrocyte sedimentation rate and a C-reactive
protein, and a laboratory documentation of N. gonorrhoeae
or C. trachomatis. An ectopic pregnancy usually presents
with abdominal pain, abnormal bleeding, and amenorrhea. A
pregnancy test should be the next test to rule out an
ectopic pregnancy. Also, even though her symptoms are more
consistent with PID, because she is not sure of her LMP
date, a pregnancy test is indicated at this time.
A pregnancy test should be performed to rule out an
ectopic pregnancy before treatment for suspected PID is
given. Also, admission and intravenous therapy (choice A)
for all adolescents with PID used to be recommended, but
this is no longer true because there is not a lot of data
to support this recommendation. However, admission is
recommended for the following individuals, including
noncompliant patients, pregnant patients with PID, an
abscess or peritonitis, HIV patients with PID, and all
those that fail outpatient therapy.
A culdocentesis (choice B) is not the most appropriate
next step at this time. This procedure, which is
48
transvaginal sampling of the cul-de-sac, may be performed
to support the diagnosis of an ectopic pregnancy if
clotted blood is found. It may also be used to obtain
bacteriologic samples. It is not indicated at this time
because there is a cervical discharge present that is sent
to pathology for evaluation, and a urine pregnancy test
must first be done to evaluate for pregnancy.
If this patient has an uncomplicated case of PID (without
a tuboovarian abscess), prescribing ofloxacin and
metronidazole therapy, orally, and discharging her (choice
D) will be the most likely treatment plan. A pregnancy
test must first be performed to rule out an ectopic
pregnancy.
If this patient has PID, it is appropriate to evaluate and
treat sexual partners (choice E). However, since this
patient is unsure of the date of her last LMP, a pregnancy
test is the most appropriate next step. Contacting sexual
partners should wait until therapy for this patient is
initiated.
You are called to the delivery room after a full-term male infant is born
via cesarean section to a G2P1 mother. Under the radiant warmer, the baby
is crying and has a heart rate of 90/min. There is some flexion of the
extremities and he grimaces when the catheter is passed in the nostril. T
baby's body is pink, but the extremities are blue. The baby's Apgar score
at 1 minute is
A.
B.
C.
D.
E.
5
6
7
8
9
Explanation:
The correct answer is B. APGAR is a named after Virginia Apgar, M.D. who
came up with a rating system for neonates at delivery. One helpful
mnemonic is A: appearance, P: pulse, G: grimace or response to catheter
nostril, A: activity or tone, and R: respiratory effort. Each category i
rated from 0-2. This child has an apgar of 6.
The remainder of the choices are incorrect as you can see from the chart
49
A 21-year-old woman comes to the student health clinic
complaining of “painful periods” for the past few years.
She describes the pain as moderate to severe, crampy in
nature, and located in her lower abdomen. She has never
been sexually active because, as she tells you, she is
waiting until she is married. The patient's physical and
pelvic examinations are normal as is a Pap smear, which
returns 5 days later. The most appropriate next step in the
management of this patient is to
A. do a progesterone withdrawal test
B. measure β-Human Chorionic Gonadotropin (β-HCG)
levels
C. perform a dilatation and curettage
D. recommend a therapeutic trial of ibuprofen
E. refer her to a psychiatrist
F. send her for a diagnostic laparoscopy
Explanation:
The correct answer is D. This patient is presenting with
classic symptoms of dysmenorrhea, perhaps the most common
complaint of gynecologic patients. The pain may precede
menses, but typically has its onset with the first day of
flow, and subsides after few days. Most women report
relief of symptoms with antiprostaglandin medications such
as ibuprofen. Heating pads and hot water bottles applied
to the lower abdomen are sometimes helpful as well.
The progesterone withdrawal test (choice A) is used in the
evaluation of amenorrhea. It has no role in the diagnosis
or treatment of dysmenorrhea.
There is no indication that this patient may be pregnant
(choice B). She is reporting normal menstrual cycles.
Measuring a β-HCG level at this time would not be
informative and would not affect the treatment of her
dysmenorrhea.
Dilatation and curettage (choice C) is not the most
appropriate next step for this patient. This is a
procedure that is most commonly used in women with
postmenopausal bleeding. It has no role in the treatment
of dysmenorrhea.
Referral to a psychiatrist (choice E) is not appropriate
at this time. The patient should be started on a
therapeutic trial of ibuprofen initially. If the patient
continues to complain of pain out of proportion to her
physical exam and other diagnostic findings, a psychiatric
referral can be considered.
A diagnostic laparoscopy (choice F) is not the most
appropriate next step for this patient. It is a procedure
that can be used to directly visualize ectopic endometrial
tissue, if endometriosis is suspected. Other symptoms
associated with endometriosis include dyspareunia and pain
with defecation. Physical exam often reveals multiple
50
tender nodules along the uterosacral ligament during a
rectal-vaginal examination.
A 20-year-old woman comes to the office with a 4-month
history of "missed periods." Prior to these past 4 months
she says that she had normal menses, which began when she
was 14 years old and occurred every 29 days. She has had
"occasional" sexual experiences over the past few years and
admits to a total of 4 sexual partners. She and her
partners always use condoms for contraception. She recently
started a new job that is "stressing her out completely"
and her sleep habits have been "a mess." Her temperature is
37.0 C (98.6 F), blood pressure is 110/70 mm Hg, pulse is
73/min, and respirations are 11/min. Her physical
examination, including a pelvic examination, is
unremarkable. The most appropriate next step is to
A. determine beta hCG level
B. obtain cortisol level
C. obtain FSH level
D. order prolactin level
E. order testosterone level
F. reassure her that it is most likely due to
stressors in her life
Explanation:
The correct answer is A. The most common cause of
amenorrhea in women of childbearing age is pregnancy.
Despite the fact that this patient she "always" uses
condoms during sexual activity, the most appropriate first
test would be a serum beta hCG to rule out pregnancy.
Elevated levels of cortisol (choice B) can suppress
gonadotropins and therefore glucocorticoid excess may be a
cause of amenorrhea. It is appropriate to evaluate for
Cushing's syndrome if there are any signs of cortisol
excess. Adrenal insufficiency may also cause amenorrhea
and a cosyntropin stimulation test may be performed if
clinically indicated by signs of adrenal insufficiency
such as hypotension, hyperkalemia, hyponatremia, etc.
FSH levels (choice C) are helpful in the evaluation of
amenorrhea. A normal FSH suggests a uterine etiology. A
low FSH suggests a pituitary etiology for amenorrhea. A
high FSH suggests ovarian failure as the etiology of
amenorrhea.
Prolactin (choice D) elevations are a common cause of
amenorrhea. It should be part of the secondary evaluation
of amenorrhea if pregnancy tests are negative. If levels
are elevated, an MRI of the pituitary should follow.
Elevated testosterone (choice E) may cause amenorrhea and
is most typically elevated in polycystic ovary syndrome.
Testing is indicated if amenorrhea is accompanied by
51
virilization.
While stress and lifestyle changes can cause abnormalities
in the menstrual cycle (choice F), 4 months of amenorrhea
should be evaluated with a pregnancy test since the most
common cause of amenorrhea in a woman of childbearing age
is pregnancy.
A 22-year-old pregnant woman is admitted to the hospital
for evaluation of severe headaches. She is 20-weeks
pregnant and has had a progressively worsening nonpulsatile
headache for the past 4 days. Vital signs at the time of
admission are: temperature 37.0 C (98.6 F), blood pressure
120/67 mm Hg, and pulse 88/min. Physical examination is
normal. A few hours after admission, the patient has a
witnessed seizure in which she has several minutes of
abnormal tonic-clonic movements. A CT scan of the head is
performed and demonstrates cerebral edema within the right
temporal lobe and deep gray matter. In addition, there is
hyperattenuation within the region of the superior sagittal
sinus. A small foci of petechial hemorrhage is also seen in
the right temporal lobe. The most appropriate management of
this patient is to
A.
B.
C.
D.
E.
administer heparin
administer warfarin
induce labor
order an MRI of the brain
perform a lumbar puncture for CSF analysis
Explanation:
The correct answer is A. Patients with dural sinus and
cerebral venous thrombosis can present with headaches,
stroke-like symptoms, and/or seizures. There are multiple
etiologies including dehydration, sepsis, and trauma. In
addition, hypercoagulable states such as sickle cell
disease, leukemia, and pregnancy are also predisposing
factors. Because the symptomatology of a dural sinus
thrombosis is nonspecific, imaging studies play a critical
role in the diagnosis. On non-contrast enhanced head CT, a
cord sign or tubular hyperdensity may be seen in the acute
setting along with cortical and subcortical hemorrhage due
to venous stasis and infarction. The treatment of choice
during pregnancy is intravenous heparin. Unfractionated
heparin and low molecular weight heparin (LMWH) do not
cross the placenta and are considered safe for the fetus,
but must be administered parenterally.
The administration of warfarin (choice B) is
contraindicated during pregnancy. Warfarin crosses the
placenta and can cause bleeding and birth defects. Heparin
is the anticoagulant of choice during pregnancy because it
52
does not cross the placenta.
The induction of labor (choice C) is not indicated in this
patient. She is only 20-weeks pregnant and is not ready to
deliver. Furthermore, induction of labor is not necessary
in the treatment of dural sinus and cerebral venous
thrombosis.
An MRI of the brain (choice D) is not necessary to make
the diagnosis of dural sinus thrombosis in this patient.
The CT scan of the head already demonstrates findings
consistent with thrombosis of the superior sagittal sinus
as well as hemorrhage due to venous stasis. If the CT scan
of the head is non-diagnostic, an MRI of the brain or a
cerebral angiography can be performed for further
evaluation.
A lumbar puncture for CSF analysis (choice E) is not
necessary for this patient's diagnosis or treatment. The
symptoms and CT findings are consistent with dural sinus
thrombosis. CSF analysis is useful in patients who are
suspected of having subarachnoid hemorrhage or meningitis.
It has no role in dural sinus thrombosis.
A previously healthy 3-week-old baby is brought by his
parents to your emergency department with a 1-day history
of emesis. The parents describe the emesis as "forceful",
non-bloody, and non-bilious. The baby is exclusively
breastfed and continues to be hungry after each episode of
vomiting. They deny any fevers. You notice an active baby
boy with unremarkable vital signs. Physical examination is
significant for a peristaltic wave on the abdomen and a 2x2
cm firm mass palpated in the midepigastric region.
Laboratory studies show a bicarbonate level of 18 mEq/L.
The most likely diagnosis is
A.
B.
C.
D.
E.
annular pancreas
gastroesophageal reflux
intussusception
malrotation with volvulus
pyloric stenosis
Explanation:
The correct answer is E. This patient presents with a
classic case of pyloric stenosis. It is the most common
surgical condition seen in the newborn period. It is more
common in males than females. It is caused by a
hypertrophic pyloric muscle, which causes obstruction of
the gastric outlet. This hypertrophic muscle can be
palpated in the midepigastric region and represents the
classic "olive" associated with pyloric stenosis. The
emesis is non-bilious because the obstruction is proximal
to the ligament of Treitz. Once the diagnosis is made,
53
treatment consists of a pyloromyotomy.
Annular pancreas (choice A) is caused by the failure of
the resorption of the left pancreatic bud during rotation
of the duodenum during fetal development, which causes a
ring-like structure around the duodenum, often the second
portion. These patients present with signs of partial or
complete small bowel obstruction and will have frequent
bilious vomiting and abdominal distension often in the
first week of life. There is often a history of
polyhydramnios.
Patients with gastroesophageal reflux (choice B) rarely
present with projectile vomiting. Parents often complain
of spitting up with feeds, arching of the back, and waking
up from sleep crying. Mild GER often responds to changing
to an upright feeding position and thickening the feeds
with cereal. Moderate GER may require medication, such as
an H2-blocker, for control. Severe reflux may require
surgical intervention.
Patients with intussusception (choice C), caused by an
invagination of the bowel, classically present with
colicky abdominal pain, raising of their knees to their
chest to relieve this pain, a sausage-like abdominal mass,
and "current jelly" stools. They may also have vomiting
that begins as non-bilious and progresses to bilious as
the obstruction progresses.
Malrotation with volvulus (choice D) is caused by failure
of the small intestine and right colon to reach their
proper anatomic location during fetal development and
twisting of the gut, resulting in compromised perfusion to
this area. These patients have bilious vomiting, may have
bloody stools, and appear quite ill as compromised
perfusion progresses to infarction and gangrene.
A 57-year-old gravida 3, para 3 woman comes to the
emergency department with abdominal discomfort and says she
feels like she is "bloated". She denies any nausea or
vomiting and has had regular bowel movements, but has lost
over 15 lbs over the last year unintentionally. Her past
medical history and surgical history are unremarkable. Her
family history is significant for diabetes and colon
cancer. She has smoked half a pack of cigarettes a day for
over 20 years, but denies alcohol or drug use. Her vital
signs are: temperature of 37.0 C (98.6 F), blood pressure
of 137/76 mm Hg, and pulse of 83/min. Physical examination
shows abdominal distension and diffuse abdominal pain, but
no rebound tenderness or guarding. All of her laboratory
studies are within normal range. A transvaginal ultrasound
shows a complex left adnexal mass with a solid and cystic
component measuring 4 cm by 4 cm in diameter. Besides the
appearance of the mass, the other feature that would be
helpful in detecting an early malignant ovarian tumor is
54
A. differences in temperature of tumor tissue
B. discordance of ovarian artery blood supply from
left to right ovaries
C. increased blood flow of ovarian arteries
D. neovascularity of tumor blood supply
E. ultrasonographic pattern of tissues surrounding
the ovaries
Explanation:
The correct answer is D. The most promising technique for
ovarian cancer screening and an important adjunct for
radiologists in establishing a mass as possibly malignant,
is through the use of transvaginal ultrasound (TV U/S)
with Doppler color flow imaging. Angiogenesis, the
formation of new blood supply, is essential in any tissue.
In order for the growing carcinoma to survive, new
microvessels have to be established to provide oxygen and
nutrients, and to remove waste products. TV U/S on Doppler
mode can detect blood movement, "ectopic blood flow
patterns," in these newly formed vessels and can help to
detect ovarian tumors at early stages.
Differences in temperature of tumor tissue (choice A) is
incorrect. There are no significant differences in
temperature between malignant and benign tissues.
Discordance of ovarian artery blood supply from left to
right ovaries (choice B) is incorrect because differences
of blood flow from one side to the other are very
unreliable and certainly not generally useful.
Increased blood flow of ovarian arteries (choice C) is
incorrect. Blood flow patterns of ovarian arteries differ
from one person to the next. Additionally, increased blood
flow of the ovarian arteries feeding a carcinoma are not
necessarily true, especially in an early malignant tumor.
The ultrasonographic pattern of tissues surrounding the
ovaries (choice E) is incorrect because a tumor would have
to be very large, and thus not in its early stage, in
order to distort the surrounding tissues. Appearance of
the surrounding tissue is generally not useful in
predicting malignant versus benign tissue.
A 37-year-old woman comes to the clinic because of severe
headaches. She is 38-weeks pregnant and has had headaches
on and off throughout her pregnancy. In the past week the
headaches have progressively worsened. Vital signs are:
temperature 37.0 C (98.6 F), pulse 70/min, and blood
pressure 180/100 mm Hg. Physical examination reveals
moderate pitting edema in both lower extremities.
Neurologic examination is normal. Fetal heart rate
monitoring is performed in the office and it demonstrates a
baseline heart rate of 120/min with variable decelerations
55
to 70-80/min. The most appropriate management at this time
is to
A.
B.
C.
D.
blood
E.
admit her to hospital for induction of labor
do a complete fetal sonographic survey
order a CT scan of the head
order serum liver function tests and a complete
count
order a urinalysis to evaluate for protein
Explanation:
The correct answer is A. This patient has signs and
symptoms of preeclampsia, which is a syndrome of high
blood pressure during pregnancy associated with
proteinuria. The complications of preeclampsia include
placental abruption, premature labor, and fetal distress.
Fetal heart tracing in this case, demonstrates variable
decelerations which indicate fetal distress. Giving birth
is the only cure for preeclampsia. Factors that influence
the decision to induce labor or perform a cesarean
delivery, include fetal distress, age of the fetus, and
severity of the condition.
A fetal sonographic survey (choice B) can be performed for
multiple reasons. It is often used to detect fetal
anomalies and to evaluate for appropriate development.
However, this study is not necessary at this time. Signs
of fetal distress in the setting of preeclampsia warrant
admission to the hospital for a possible induction of
labor or cesarean delivery.
A CT of the head (choice C) is not warranted in this
patient. Complications of preeclampsia include cerebral
infarct and hemorrhage. This patient has no neurologic
findings to suggest these complications and therefore, a
CT is not necessary.
Serum liver function tests and a complete blood count
(choice D) are not critical in addressing the urgent issue
of fetal distress. A complication of preeclampsia is HELLP
syndrome. This syndrome manifests as hemolysis, elevated
liver enzymes, and low platelet count. If this syndrome is
suspected, the serum liver function tests and complete
blood count are necessary to make the diagnosis.
A urinalysis to evaluate for protein (choice E) is not the
most appropriate next step in the management of a
pregnancy with signs of fetal distress. Proteinuria is a
sign of preeclampsia, and a urinalysis is often ordered in
pregnant women with high blood pressure. This type of work
up should be pursued in patients who are less critically
ill and who do not have pregnancies complicated by fetal
distress.
A 26-year-old financial analyst comes to the office for her
56
28-week prenatal visit. You have followed her since her 12week visit and her pregnancy course has been complicated
only by some nausea and vomiting late in the first
trimester. She is up to date on all of her laboratory work,
and an 18-week anatomy scan was within normal limits. She
has had appropriate weight gain and continues with prenatal
vitamins and iron. She has no medical problems, and does
not smoke cigarettes, drink alcohol, or participate in any
drug use. At today's visit she is complaining of some
increased fatigue and dependent edema. Her blood pressure
is 90/50 mm Hg, fundal height is appropriate and a Doppler
of the fetal heart reveals a fetal heart rate of 140 beats
per minute. At today's visit, the most important thing for
you to do is
A.
B.
C.
D.
E.
check a blood type
check a thyroid function panel
do a vaginal exam for baseline cervical exam
order a follow-up growth scan
order weekly non-stress tests
Explanation:
The correct answer is A. This patient is at her 28-week
visit. At this visit it is imperative to check 2 things: a
glucose tolerance test for gestational diabetes if the
patient has not already had one, and the patient's blood
type to target Rh-negative patients. At the 28-week visit,
all Rh-negative patients need to receive rhoGAM in order
to prevent alloimmunization and future hydrops.
There is no need to check a thyroid function panel (choice
B). In those patients with underlying thyroid disease
(hyper or hypothyroidism), a panel should be checked every
trimester. However, this patient has no history of thyroid
abnormality.
There is no need to do a vaginal exam (choice C) at this
time. The patient has no history of cervical incompetence
and gives no history of contractions or preterm labor. The
cervix is checked at a the initial prenatal visit and then
again if there are any indications of labor.
There is no need to have follow-up growth ultrasounds
(choice D) on routine prenatal patients. Twin gestations
should have growth scans to assure concordant growth. In
addition, some physicians advocate growth scans for
diabetic patients at 36 weeks in order to target
macrosomia and possible shoulder dystocia.
There is no need to order non-stress tests (choice E) in
routine prenatal patients until they reach post-dates
(after 40 weeks gestation).
A 1-day-old male neonate, born by caesarean section, at 35weeks gestation, is in the neonatal intensive care unit. He
57
has a temperature of 38.8 C (101.8 F) at the time of birth.
Physical examination reveals a small but alert boy. The
lungs are clear to auscultation. The abdomen is normal
without distension. A urinalysis reveals:
A renal ultrasound is performed because of concern for
urinary obstruction. The ultrasound reveals normal kidneys,
but there are bilateral adrenal masses consistent with
hemorrhage. Management of this patient should consist of
A. ampicillin plus gentamicin
B. ampicillin plus gentamicin and percutaneous
nephrostomies
C. ampicillin plus gentamicin and a repeat ultrasound
in 1 week
D. gentamicin and percutaneous nephrostomies
E. a voiding cystourethrogram immediately
Explanation:
The correct answer is C. This neonate has both a urinary
tract infection and adrenal hemorrhages. A urinary tract
infection in the neonate is treated with intravenous
ampicillin and gentamicin. An ultrasound is performed to
rule out gross anomalies or obstruction. In this case, an
incidental note is made of adrenal hemorrhage which is
seen almost exclusively in newborns and is an indicator of
birth trauma, stress, anoxia, or dehydration. To ensure
that these adrenal masses really are hematomas, a followup ultrasound must be performed in 1-2 weeks. A voiding
cystourethrogram (VCUG) will be necessary later to
evaluate this boy for reflux.
Ampicillin plus gentamicin (choice A) is the antibiotic
regimen of choice to treat a urinary tract infection in
the neonate. However, the adrenal hemorrhage must also be
followed up to exclude adrenal hypertrophy or other
adrenal masses.
Ampicillin plus gentamicin is the antibiotic regimen of
choice to treat a urinary tract infection in the neonate.
The adrenal hemorrhage must also be followed up to exclude
adrenal hypertrophy or other adrenal masses. Percutaneous
nephrostomies (choice B) are not indicated, as there is no
anatomic obstruction.
Gentamicin (choice D) alone is insufficient antibiotic
coverage for a urinary tract infection in the neonate.
58
Moreover, percutaneous nephrostomies are not indicated, as
there is no anatomic obstruction.
A voiding cystourethrogram (choice E) is not appropriate
at this time. Immediate treatment with antibiotics is
necessary. Secondarily, the putative adrenal hemorrhages
must also be reevaluated with a follow-up ultrasound.
You are asked to see a 3-week-old infant in the emergency
department with a 1-day history of fever. The parents
measured his temperature because he "felt warm" to them and
found a temperature of 38.3 C (101.0 F). He has been
feeding normally, taking 2 ounces of formula every 3-4
hours. He had 6 wet diapers the previous day. Examination
shows an active infant with a temperature of 38.8 C (101.8
F). His skin perfusion is good and his physical
examination, including examination of his tympanic
membranes, is normal. There are no ill household contacts.
The most appropriate next step is to
A. discharge the patient with close outpatient follow
up
B. inquire about the mother's group B streptococcal
status at delivery
C. obtain the infant's vaccination history
D. order a urinalysis and, if negative, do blood and
CSF cultures
E. send blood, urine, and CSF cultures and begin
empiric intravenous antibiotic therapy
Explanation:
The correct answer is E. There is absolutely no way to
reliably distinguish a self-limited viral illness from
sepsis or meningitis in an infant less than 4-weeks of
age. Accordingly, all infants with a fever greater than 38
C (100.4 F) in this age group require full evaluation,
including admission and parenteral antibiotics. The
incidence of sepsis is somewhere between 5-10% with this
degree of fever and can be catastrophic if missed,
resulting in death or permanent neurologic disability.
Discharging the patient (choice A) is inappropriate
because well-appearing infants in this age group may still
have a potentially lethal bacterial disease.
At 3 weeks, this infant is at the peak incidence of late
onset group B streptococcal disease, but the mother's
group B streptococcal status (choice B) does not correlate
well with occurrence of late onset group B streptococcal
disease and so it is not relevant here. Maternal group B
streptococcal screening is useful in the management of
early-onset disease (disease within the first few days of
life).
59
At 3 weeks this infant is unlikely to have received any
vaccinations (choice C), with the possible exception of
hepatitis B vaccine. He would not in any case have
received Haemophilus influenzae or conjugated
Streptococcus pneumoniae vaccine, which are the only two
that could potentially influence his susceptibility to
sepsis or meningitis.
Urinalysis (choice D) is not reliable in excluding urinary
tract infection in very young infants, and therefore is
not a good screening test to decide if further evaluation
is necessary.
You are notified that one of your patients, a 35-year-old
pregnant woman, has gone into labor at 39 weeks. By the
time you arrive at the hospital, the baby has already been
born during an uneventful delivery, about 2 minutes
earlier. You go to examine the baby on the warmer and
discover that he has respiratory distress and is becoming
cyanotic. You attempt to oxygenate him with a mask, but
this does not seem to provide him with much relief.
Physical examination shows subcostal and intercostal
retractions, absent air entry on the left side, poor air
entry on the right side, and "gurgle-like" sounds in the
left chest. The heart sounds are best heard in the right
hemithorax; the abdomen is flat without organomegaly. The
most likely cause of these findings is
A. aspiration of meconium at delivery
B. a defect in the left hemidiaphragm
C. a deficiency of surfactant
D. dextrocardia and situs inversus
E. esophageal atresia with distal tracheoesophageal
fistula
F. transposition of the aorta and pulmonary artery
Explanation:
The correct answer is B. This patient most likely has a
congenital diaphragmatic hernia, which is when the
abdominal contents herniate into the left hemithorax
through a congenital defect in the left hemidiaphragm.
This causes displacement of the heart into the right
hemithorax and pulmonary hypoplasia. If gas has entered
the bowel tract, bowel sounds (gurgle-like noises) can be
heard in the chest. The displacement of the abdominal
contents renders the abdomen flat. In cases of severe
pulmonary hypoplasia, administration of oxygen results in
only a poor improvement in oxygenation.
Meconium aspiration (choice A) should be accompanied by a
history of fetal stress, meconium stained amniotic fluid,
or meconium staining of the skin, placenta, or umbilical
60
cord. Physical findings are symmetric, with no
displacement of the heart, the breath sounds should be
similar on both sides, and there should not be bowel
sounds in the chest.
Hyaline membrane disease is caused by a deficiency of
surfactant (choice C) and is a rare condition in an infant
born at 39 weeks. It typically occurs in infants born
before 32 weeks, when surfactant is produced. Surfactant
is a phospholipid that prevents alveolar collapse by
increasing lung compliance and decreasing surface tension.
It is very unlikely that this patient has dextrocardia and
situs inversus (choice D), which are rare positional
anomalies where the apex of the heart is on the right
(instead of the left) and the abdominal viscera are
reversed (the liver is on the left and the stomach is on
the right). In cases of dextrocardia and situs inversus,
the breath sounds should be basically normal on both
sides, and you should not be able to hear bowel sounds in
the chest.
Esophageal atresia with distal tracheoesophageal fistula
(choice E) usually presents with a history of
polyhydramnios, cyanosis with feeding, and increased
oropharyngeal secretions. It is unusual for this condition
to present so soon after birth with severe respiratory
distress.
Transposition of the aorta and pulmonary artery (choice F)
is a rare form of congenital heart disease where the aorta
arises from the right ventricle and the pulmonary artery
arises from the left ventricle. Infants with this disorder
are cyanotic from birth and congestive heart failure
usually follows. However the breath sounds would be
similar on both sides and there would not be bowel sounds
in the chest.
61